• Shuffle
    Toggle On
    Toggle Off
  • Alphabetize
    Toggle On
    Toggle Off
  • Front First
    Toggle On
    Toggle Off
  • Both Sides
    Toggle On
    Toggle Off
  • Read
    Toggle On
    Toggle Off
Reading...
Front

Card Range To Study

through

image

Play button

image

Play button

image

Progress

1/159

Click to flip

Use LEFT and RIGHT arrow keys to navigate between flashcards;

Use UP and DOWN arrow keys to flip the card;

H to show hint;

A reads text to speech;

159 Cards in this Set

  • Front
  • Back

381. Which one of the following is NOT considered an agent that can exacerbate effects in the irradiated lung?
A. Busulfan
B. Cyclophosphamide
C. Gamma interferon
D. Vincristine


Answer 381 is D. Vincristine has no known significant lung toxicity.


382. Which one of the following is the CORRECT staging for lung adenocarcinoma with ipsilateral satellite tumor nodules within the primary tumor lobe and metastatic adenopathy to the ipsilateral supraclavicular lymph nodes?
A. T3 N2 M0
B. T4 N2 M0
C. T3 N3 M0
D. T4 N3 M0


Answer 382 is D. According to AJCC staging, T4 N3 M0 is the correct designation.


383. Recursive partitioning analysis of RTOG data in non small cell lung cancer (NSCLC) identified five subgroups of patients with significantly different median survival times. Which one of the following carries the lowest median survival?
A. Karnofsky performance status (KPS) equal or greater than 90% with chemotherapy
B. KPS less than 90% with age less than 70 and non large cell histology
C. Age older than 70 with pleural effusion
D. KPS equal or greater than 90% with pleural effusion

Answer 383 is C. Age >70 with pleural effusion has a median survival time of 2.9
months. Median survival times for choices A, B and D are 16.2 months, 9.7 months and
6.1 months, respectively. (Werner-Wasik M, Scott C, Cox JD, et al. Recursive
partitioning analysis of 1999 Radiation Therapy Oncology Group (RTOG) patients with
locally-advanced non-small-cell lung cancer: identification of five groups with different
survival. Int J Radiat Oncol Biol Phys. 2000;48:1475-1482.)

384. Which one of the following paraneoplastic syndromes is associated exclusively with small cell lung cancer?
A. Gynecomastia
B. Hypercalcemia of malignancy
C. Hypertrophic pulmonary osteoarthropathy
D. Inappropriate ADH

Answer 384 is D. Gynecomastia is seen more commonly with large cell histology.
Hypercalcemia of malignancy is seen more commonly with squamous cell carcinoma
histology. Hypertrophic pulmonary osteoarthropathy is more commonly seen with
adenocarcinoma histology.

275. Base transversion mutations, identified by next-generation sequencing in cells from a small cell lung carcinoma, are caused by which of the following?
(A) Radon
(B) Tobacco
(C) Mutant EGFR
(D) Asbestos

Key: B
Rationale: Base transversion mutations, such as G>T or C>A, are associated with tobacco smoke carcinogens, a risk factor for the development of small cell lung carcinoma. Several recent articles have highlighted the power of next-generation sequencing methods to analyze the genomic profile of cancer cells.
References: Pleasance et al., Nature 2011; 463:184-90; Cassidy and Venkitaraman, Curr Op Genet Dev 2012; 22:10-13.

298. The randomized clinical trial comparing thoracic radiation of 45 Gy given in once-daily to twice-daily in addition to current chemotherapy for limited-stage SCLC demonstrated:
(A) 10% 5-year survival benefit with twice-daily radiation.
(B) Similar 5-year survival rates for both radiation regimens.
(C) 60% local failure rate with twice-daily radiotherapy.
(D) Significantly worse long term toxicity with twice-daily radiation.

Key: A
Rationale: Four cycles of ciplatin plus etoposide and a course of radiotherapy (45 Gy, given either once or twice daily) beginning with cycle 1 of the chemotherapy resulted in 5-year survival rates of 26% for patients receiving twice-daily radiotherapy vs 16% for patients receiving once-daily radiotherapy, for limited-stage small-cell lung cancer.
References: Turrisi AT, Kim K, Blum R, et al. Twice-daily compared with once-daily thoracic radiotherapy in limited small-cell lung cancer treated concurrently with cisplatin and etoposide. N Engl J Med 1999; 340:265-271.

386. Limited-stage small-cell lung cancer is defined as disease confined to which one of the following?
A. The thorax, ipsilateral as well as contralateral hemithorax, which can be covered within a single radiation port
B. A hemithorax with contralateral hilar disease, which can be covered within a single radiation port
C. A hemithorax with contralateral hilar/mediastinal disease, which can be covered within a single radiation port
D. A hemithorax with contralateral hilar/mediastinal/supraclavicular disease, which can be covered within a single radiation port
E. An ipsilateral hemithorax, which can be covered in a single radiation port

Answer 386 is E. This arose from the early Veteran’s Administration Lung Study Group system, which defines “Limited Stage” SCLC as disease confined to one hemithorax, potentially encompassed in a tolerable radiation field.

333. According to the original VA definition, which of the following patient presentations would be classified as limited-stage small cell lung cancer?
A. A 3-cm left upper-lobe lung tumor and a right hilar lymph node
B. A 3-cm left lower-lobe tumor with a malignant pleural effusion
C. A 7-cm right upper-lobe lung tumor with a right hilar lymph node
D. A 7-cm right upper-lobe lung tumor with a right anterior cervical lymph node

Answer 333 is C. Ref: NCCN 2005 Practice Guideline, ST-1 Limited stage is the disease confined to the ipsilateral hemithorax within a single radiation port.

337. Which of the following statements about prophylactic cranial irradiation (PCI) for patients with small cell lung cancer is true?
A. It may be considered for patients with a complete response to treatment.
B. It should be delivered concurrently with chemotherapy.
C. It is commonly administered at 2 Gy per fraction to 40 Gy in 4 weeks.
D. There is no decrease in CNS failure for patients who receive PCI.

Answer 337 is A. Ref: 2005 NCCN Practice Guidelines, MS10-11.

340. Which of the following risk factors for the development of mesotheliomas is most significant?
A. Asbestos exposure
B. Chronic lung infection
C. Cigarette smoking
D. Radiation

Answer 340 is A. There is clear relationship between asbestos exposure, smoking and lung cancer, but the relationship between smoking and mesotheliomas is not clear.

342. Which of the following techniques is NOT appropriate for accurately staging non-small cell lung cancer?
A. Chamberlain procedure for right-sided lesions and superior mediastinum
B. Video-assisted thoracoscopy
C. CT scan of the lungs and upper abdomen
D. PET scan

Answer 342 is A. Chamberlain’s procedure is used for AP window and prevascular lymph nodes from left sided tumors.

347. Which of the following statements about radiation-induced lung injury is FALSE?
A. Pneumonitis occurs between 1 and 6 months after radiation therapy.
B. Patients may present with cough, shortness of breath, and mild fever.
C. Rapid tapering of steroid therapy is recommended.
D. SPECT perfusion and ventilation scans are more sensitive than CT scans in detecting it.

Answer 347 is C. A slow taper of steroids is necessary or symptoms may rebound. Infection does need to be ruled out if patient does not improve.

11. What is the second leading cause of lung cancer?
A. Exposure to arsenic
B. Exposure to asbestos
C. Exposure to benzene
D. Exposure to radon

Correct answer is D. RATIONALE: Radon is the second leading single agent responsible for causing lung cancer; asbestos is responsible for only 3% to 4% of cases.

23. Which of the following statements about video-assisted thoracic surgery for patients with lung cancer is FALSE?
A. The postoperative period is shorter.
B. It decreases the risk of intraoperative bleeding.
C. It is a viable alternative to thoracotomy in patients who are frail.
D. Locoregional recurrence is increased.

Correct answer is D. RATIONALE: The regional recurrence and long-term survival with video-assisted thoracic surgery (VATS) and lymph node dissection are equal to open lung resection.

37. Which of the following statements about adenocarcinoma of the lung is true?
A. It is the most common histologic subtype.
B. It is treated differently than squamous cell carcinoma.
C. It is less likely to metastasize to the brain than other histologies.
D. It is rarely associated with TTF-1.

Correct answer is A. RATIONALE: Adenocarcinoma has surpassed squamous cell carcinoma of the lung as the most common histological subtype.


62. Which of the following statements about the use of amifostine concurrently with thoracic external-beam radiation therapy is true?
A. Pneumonitis is decreased.
B. Esophagitis is increased.
C. Xerostomia and mucositis are increased.
D. Normal lung cells do not take up amifostine.

Correct answer is A. RATIONALE: Amifostine treatment before lung irradiation protects the lung alveolocapillary integrity and has been found to have a protective effect in the acute phase of radiation lung injury.

74. Which of the following treatment techniques is NOT an effective method for providing expedient palliation for an endobronchial lesion?
A. Photodynamic therapy
B. 125I LDR therapy
C. 192Ir HDR therapy
D. Endobronchial stent placement

Correct answer is B. RATIONALE: 125I LDR therapy is not an effective method for providing expedient palliation for an endobronchial lesion.

88. What effect does external-beam radiation therapy have on lung function?
A. Increases TLC
B. Increases DLCO if chemotherapy is added
C. Decreases FEV1/VC ratio
D. Decreases DLCO

Correct answer is D. RATIONALE: Both radiation therapy alone and in combination with chemotherapy cause an immediate decline in DLCO (corrected or uncorrected) starting within 1 month after radiation therapy with a nadir at 9 months.

102. Which of the following statements about staging for non-small cell carcinoma of the lung is true?
A. CT scan provides approximately 95% sensitivity and specificity.
B. Screening tests have reduced the mortality rate for high-risk patients.
C. PET scanning has replaced mediastinoscopy for determining cancer staging.
D. Positive PET scan findings require pathological confirmation.

Correct answer is D. RATIONALE: Mediastinoscopy is still considered the gold standard for staging.

114. What stage of non-small cell lung cancer is most appropriate for a patient who has a 3-cm mass in the right upper lobe, mediastinal adenopathy, and a 2-cm nodule in the right lower lobe of the lung?
A. Stage IIB
B. Stage IIIA
C. Stage IIIB
D. Stage IV

Correct answer is D. RATIONALE: AJCC Cancer Staging Manual.

130. What is the best treatment for malignant mesothelioma?
A. Extrapleural pneumonectomy, followed by chemotherapy and adjuvant radiation therapy to a dose of 50 Gy to 60 Gy
B. Pleurodesis, followed by systemic chemotherapy with subsequent external-beam radiation therapy to a dose of 60 Gy in 20 fractions
C. IMRT to a localized positive margin to a radiation dose of 70 Gy
D. IMRT to the entire lung to a radiation dose of 45 Gy

Correct answer is A. RATIONALE: For healthy patients with malignant mesothelioma, the best results have been achieved with trimodality therapy.

145. Which of the following statements about asbestos exposure and mesothelioma is true?
A. The carcinogenic effects appear to result from its physical properties rather than its chemical structure.
B. Mesothelioma cell lines appear to be less sensitive than non-small cell lung cancer cell lines to radiation.
C. Screening of asbestos workers for mesothelioma is an effective way to improve the cure rate of this disease.
D. It is rare for mesothelioma to be misdiagnosed.

Correct answer is A. Reference: Devita, VT, Hellman, S, Rosenberg, SA. Cancer Principles and Practice of Oncology. 6th edition. Chapter 40, Sec. 2.

161. Which of the following statements about malignant mesothelioma is true?
A. The median patient survival time is 24 months.
B. Most patients have bilateral disease at presentation.
C. Diagnosis is most commonly established via cytology of pleural effusions.
D. It commonly spreads along the biopsy tract.

Correct answer is D. RATIONALE: It is important to know where malignant mesothelioma spreads since these tracts have to be covered by a radiation port.

177. Which of the following statements about TNM staging for lung cancer is FALSE?
A. Stage N1 involves lymph node stations 10-14.
B. Stage N2 involves the ipsilateral mediastinal lymph nodes.
C. Stage N3 involves the contralateral hilar and mediastinal lymph nodes.
D. Stage M1 involves the contralateral supraclavicular lymph nodes.

Correct answer is D. Reference: AJCC Cancer Staging Manual.

191. Which of the following statements about malignant mesothelioma is true?
A. Chemoradiation therapy alone offers promising results.
B. Most patients die of metastatic disease and not of local recurrence.
C. Patients who have the epithelial subtype have a worse prognosis.
D. Mediastinal lymph node involvement affects the prognosis.

Correct answer is D. Reference: Journal of Clinical Oncology (JCO). 1993;11:1172-1178.

205. Which of the following conditions should be treated like small cell lung cancer?
A. Combined small cell lung cancer and non-small cell lung cancer
B. Non-small cell lung cancer with neuroendocrine features
C. Large cell neuroendocrine tumors
D. Atypical carcinoid tumors

Correct answer is A. Reference. Journal of the National Comprehensive Cancer Network (JNCCN). 2006;4:614.

221. Which of the following radiation doses administered concurrently with chemotherapy is considered to be the most INAPPROPRIATE treatment for limited-stage small cell lung cancer?
A. 45 Gy at 1.5 Gy per fraction twice daily
B. 45 Gy at 1.8 Gy per fraction
C. 50 Gy at 2.0 Gy per fraction
D. 60 Gy at 2.0 Gy per fraction

Correct answer is B. Reference. Journal of the National Comprehensive Cancer Network (JNCCN). 2006;4:613.

237. Which of the following treatments would NOT be appropriate after a lobectomy with negative margins is performed for a patient who has limited-stage small cell lung cancer?
A. Radiation therapy with positive mediastinal nodal dissection
B. Chemoradiation therapy with negative mediastinal nodal dissection
C. Chemotherapy with negative mediastinal nodal dissection
D. Chemotherapy with negative mediastinal nodal sampling

Correct answer is B. RATIONALE: There is no evidence that radiotherapy benefits the patient with a negative mediastinal nodal dissection. Reference: Journal of the National Comprehensive Cancer Network (JNCCN). 2006;4:610.

255. Which of the following conditions does NOT belong in the paraneoplastic category of immune-mediated neurological syndromes in small cell lung cancer?
A. Eaton-Lambert syndrome
B. Cerebellar degeneration
C. Hypernatremia
D. Retinopathy

Correct answer is C. Reference: Gandhi, L, Johnson, BE. Journal of the National Comprehensive Cancer Network (JNCCN). 2006;4:631-638.

273. According to the VA Lung Cancer Group's classification, which of the following sites of lymphadenopathy is associated with extensive-stage small cell lung cancer?
A. Ipsilateral hilar lymph nodes
B. Ipsilateral supraclavicular lymph nodes
C. Contralateral supraclavicular lymph nodes
D. Contralateral mediastinal lymph nodes

Correct answer is C. Reference: Journal of the National Comprehensive Cancer Network (JNCCN). 2006;4:615.

313. A 62-year-old man with limited-stage small cell lung cancer and proximal muscular weakness is most likely to have which of the following conditions?
A. SIADH
B. Myasthenia gravis
C. Cerebellar degeneration
D. Eaton-Lambert syndrome

Correct answer is D. RATIONALE: This is a typical presentation of Eaton-Lambert Myasthenic syndrome seen in about 1% to 2% of patients with small cell lung cancer. EMG and antibody testing to P/Q VGCC are diagnostic. Reference: Darnell, RB, Posner, JB. Paraneoplastic syndromes affecting the nervous system, Seminars in Oncology, 2006,33:270-298.

326. The 5-year survival for patients who have undergone radiation therapy for inoperable stage III invasive thymoma is approximately:
A. 20%.
B. 50%.
C. 70%.
D. 90%.

Correct answer is B. RATIONALE: NCI, PDQ, Invasive thymoma and thymic carcinoma, inoperable stage III and III invasive thymoma, Level of evidence, 3iiiDiii.


337. Which of the following diagnostic tests is NOT included in the staging of small cell lung cancer?
A. CT scan of the chest and upper abdomen
B. Brain imaging
C. Bone scan
D. PET scan

Correct answer is D. RATIONALE: A PET scan is considered a supplementation and not a replacement to CT scanning, brain imaging, and bone scans. The use of PET scans for diagnostic staging of small cell lung cancer is undergoing evaluation in clinical trials.

349. Which of the following types of cancer is most commonly associated with myasthenia gravis?
A. Small cell lung cancer
B. Non-small cell lung cancer
C. Mesothelioma
D. Thymoma

Correct answer is D. Reference: Engels, EA, Pfeiffer, RM. International Journal of Cancer. 2003;105(4):546-551.

281. Which of the following is true regarding RTOG 9410, a phase III trial comparing sequential to concurrent chemoradiation for patients with locally advanced NSCLC?
(A) Sequential therapy arm consisted of cisplatin and etoposide followed by radiation to 63 Gy.
(B) Concurrent therapy arm consisted of chemotherapy and hyperfractionated radiation.
(C) Five-year overall survival was significantly improved with concurrent chemoradiation.
(D) The rate of acute toxicity was the same between all arms.

Key: C
Rationale: Sequential chemotherapy was delivered using cisplatin and vinblastine, which concurrent chemoradiation used either cisplatin and vinblastine (for QD radiation) or cisplatin and etoposide (for BID radiation). OS was superior for concurrent versus sequential treatment. Acute toxicity was worse with concurrent treatment, though late effects were similar.
References: Curran, J Natl Cancer Inst 2011.

280. According to ACOSOG Z4031, which of the following is TRUE regarding staging FDG-PET for NSCLC?
(A) The accuracy is less than 30% in lesions <20 mm.
(B) The accuracy is greater than 90% in lesions >30 mm.
(C) The sensitivity is about 50%.
(D) The specificity is about 30%.

Key: D
Rationale: A prospective trial (ACOSOG Z4031) was reported at 2012 ASCO. This study evaluated the accuracy of FDG-PET in diagnosing NSCLC in 969 patients with clinical stage I (cT1-2N0M0) known or suspected NSCLC were enrolled in the study between 2004 and 2006. In this study, the accuracy of FDG-PET was less than 50% in lesions less than 20 mm, but greater than 80% in lesions greater than 30 mm. However, above 30 mm, the accuracy did not improve. The malignancy rate in patients with FDG-PET scans was 83%. The accuracy of FDG-PET to diagnose lung cancer was 73%, the sensitivity was 82%, and the specificity was only 31%. The positive predictive value was 85%, and negative predictive value was only 26%.

15. Which of the following statements about the phase III EORTC trial (08993), which
randomized patients with extensive-stage small cell lung cancer to receive prophylactic
cranial irradiation (PCI) or no PCI, is true?
A. Acute toxicity was significant among patients in the PCI arm of the study.
B. Patients received PCI after undergoing 3 cycles of chemotherapy.
C. The PCI doses ranged from 20 Gy in 5 fractions to 30 Gy in 12 fractions.
D. PCI did not significantly improve overall survival and disease-free survival.

Correct answer is C. Reference: New England Journal of Medicine. 2007;(357):664-72.

28. The paraneoplastic syndrome most commonly associated with small cell carcinoma of
the lung is:
A. Cushing syndrome.
B. Eaton-Lambert syndrome.
C. cerebellar degeneration.
D. syndrome of inappropriate antidiuretic hormone secretion (SIADH).

Correct answer is D. RATIONALE: All of the options are associated with small cell carcinoma of the lung, but syndrome of inappropriate antidiuretic hormone secretion (SIADH) is the most commonly associated paraneoplastic syndrome. Reference: Gunderson and Tepper. Clinical Radiation Oncology. 2nd edition, 2007.

36. Which of the following statements about malignant pleural mesothelioma is true?
A. Approximately 20% of patients have bilateral disease.
B. The left hemithorax is affected more often than the right hemithorax.
C. It can be difficult to differentiate from small cell carcinoma.
D. Epithelial histology is the most common subtype.

Correct answer is D. RATIONALE: The right hemithorax is affected more often than the left hemithorax (60% and 35% of cases, respectively), and 5% of malignant pleural mesotheliomas (MPMs) are bilateral. MPM can be difficult to differentiate from adenocarcinoma. Epithelial tumors constitute 40% of MPMs, sarcomatoid tumors constitute 20% of MPMs, mixed tumors constitute 35% of MPMs, and 5% of MPMs are undifferentiated subtype.

39. What percent of patients who have small cell lung cancer present with limited-stage
disease?
A. 10%
B. 15%
C. 30%
D. 45%

Correct answer is C. RATIONALE: Approximately 65% to 75% of patients present with
extensive-stage small cell lung cancer, with 10% to 15% having brain metastases at
presentation.

59. Which of the following chemotherapy agents can be delivered in full doses concurrently with thoracic radiation therapy?
A. Carboplatin
B. Gemcitabine
C. Paclitaxel
D. Cisplatin

Correct answer is D. RATIONALE: Cisplatin is the only agent that can be administered in full (systemic) doses with concurrent thoracic radiation therapy (RT) without causing undue toxicity. Both carboplatin and paclitaxel are given in attenuated (radiosensitizing) doses with thoracic RT. Gemcitabine is particularly difficult to be given concurrently with RT, due to its strongly radiosensitizing properties and resulting severe acute toxicity.

81. A 47-year-old woman who is a former smoker presents with a 3-cm mass in the left lower lobe of the lung. Bronchoscopy demonstrates a diagnosis of adenocarcinoma, and thoracic CT scan reveals a 1.5-cm subcarinal lymph node. FDG-PET scan shows hypermetabolic uptake in both the mass and the lymph node. There are no other sites of disease present. The clinical stage of this patient's disease is:
A. T1N2M0/IIIA.
B. T1N3M0/IIIB.
C. T2N2M0/IIIA.
D. T2N3M0/IIIB.

Correct answer is A. RATIONALE: A left lower lobe lung mass with a subcarinal lymph node are demonstrated on both CT and PET imaging studies. The stage is T1 (primary tumor not exceeding 3 cm) and N2 (subcarinal lymph nodes are staged as N2, even though they are in the midline of the body rather than on the ipsilateral side to the tumor). Overall the stage is IIIA, since stage IIIB requires either a T4 tumor stage or N3 lymph node stage to exist.

12. According to the latest AJCC staging manual for lung cancer, which of the following would be classified as M1a?
(A) Malignant pleural effusion
(B) Separate tumor nodule(s) in the same lobe
(C) Contralateral supraclavicular lymph node involvement
(D) Separate tumor nodule(s) in a different ipsilateral lobe

Key: A
Rationale: Pertinent changes to the lung staging system from the 6th to 7th edition include (1) more discrete stratification of tumors by size and partitioning of larger (>7 cm) tumors into T3, (2) downstaging of multiple nodules to T3 (same lobe), T4 (same lung), and M1a (contralateral lobe), and (3) upstaging of malignant pleural effusion from T4 to M1a.
References: AJCC Cancer Staging Manual, 7th edition, 2009.

89. Which of the following statements about malignant pleural mesothelioma is true?
A. It is more common in women.
B. The reported incidence has declined in the past 20 years.
C. Smoking does not increase the risk of mesothelioma in a person without prior exposure to asbestos.
D. Serpentine asbestos (chrysotile) is more carcinogenic than the rodlike amphiboles.

Correct answer is C. RATIONALE: Although reported incidence rates have increased in the past 20 years, mesothelioma is still a relatively rare cancer. Smoking does not appear to increase the risk of mesothelioma. However, the combination of smoking and asbestos exposure significantly increases a person's risk of developing cancer of the air passageways in the lung. Chrysotile is less carcinogenic than the rodlike amphiboles.

99. Which of the following statements about mediastinal germ cell tumors is true?
A. Malignant tumors are more common in men.
B. They are the most common neoplasm of the anterior mediastinum.
C. They are derived from primitive germ cells that fail to migrate during late embryonic development.
D. They rarely produce serologic markers.

Correct answer is A. RATIONALE: Malignant mediastinal germ cell tumors (GCTs) are more common in men (>90%). The most common anterior mediastinal neoplasm is thymoma (20%). GCTs are derived from primitive germ cells that fail to migrate completely during early embryonic development. These tumors often produce serologic markers, such as alphafetoprotein and human chorionic gonadotropin.

121. A 52-year-old man undergoes left lower lobe lobectomy and mediastinal lymph node dissection for early-stage squamous cell carcinoma of the lung. A 3.5-cm primary tumor with clear resection margins is found in the lobectomy specimen. Three of seven ipsilateral mediastinal lymph nodes are involved. Which of the following adjuvant therapies would be most effective for this patient?
A. Platinum-based chemotherapy for 3 cycles
B. Platinum-based chemotherapy administered concurrently with thoracic radiation therapy
C. Platinum-based chemotherapy, followed by thoracic radiation therapy
D. Thoracic radiation therapy, followed by platinum-based chemotherapy

Correct answer is C. RATIONALE: Several large phase III randomized trials (e.g., IALT study) published recently demonstrated a survival benefit for adjuvant platinum-based chemotherapy following resection of stages II-III (and possibly stage IB) non-small cell lung cancer (NSCLC). The role of adjuvant thoracic radiation therapy (RT) is less clear, although it continues to be recommended by most authorities for patients with stage III cancer. The optimal sequencing of the chemotherapy and thoracic RT remains unknown, but the current most common practice is to deliver chemotherapy first, in view of the high risk of systemic relapse in N2-positive patients, to be followed by RT (in R0 or complete resections). This sequence may be reversed in those patients who have positive microscopic resection margins (R1 or incomplete resections with microscopic residual tumor).

179. According to the phase III EORTC trial (08993), what was the 1-year survival rate for patients who received or did not receive prophylactic cranial irradiation (PCI) for
extensive-stage small cell lung cancer?
With PCI Without PCI
A. 25% 40%
B. 27% 13%
C. 33% 35%
D. 55% 25%

Correct answer is B. Reference: New England Journal of Medicine. 2007;(357):pp 664-72.

186. A phase III prospective randomized trial was conducted to study the use of chemotherapy for malignant pleural mesothelioma. What drug combined with cisplatin was found to be superior to the use of cisplatin alone?
A. Pemetrexed
B. Gemcitabine
C. Navelbine
D. Docetaxel

Correct answer is A. RATIONALE: This phase III trial demonstrated cisplatin plus pemetrexed to be superior to cisplatin alone (median survival of 12.1 months vs. 9.3 months; p=0.02). This trial resulted in the approval of the cisplatin and pemetrexed combination by the Food and Drug Association (FDA) in 2004 for the treatment of malignant pleural mesothelioma (MPM). Journal of Clinical Oncology (JCO). 2002;21:2636-44.

198. Which of the following statements about the use of radiation therapy after total resection for a Masaoka stage II thymoma is true?
A. The local failure rate after surgery alone can be as high as 30%.
B. Postoperative radiation therapy significantly reduces the rate of local recurrence.
C. A dose higher than 54 Gy is optimal for local control.
D. IMRT further improves the local control rate.

Correct answer is A. RATIONALE: Despite a high local failure rate (up to 30%) after a total resection for a stage II thymoma, the role of postoperative radiation therapy remains undefined.

210. Which of the following factors is associated with the most favorable prognosis for
patients with malignant pleural mesothelioma?
A. Nonasbestos induced
B. Epithelial histological subtypes
C. Shorter latency period after asbestos exposure
D. Female gender

Correct answer is B. RATIONALE: Malignant pleural mesothelioma (MPM) is a uniformly fatal disease. Patients with epithelial histological subtypes, negative resection margins, no lymph node involvement, and absence of local and distant metastases have the most favorable outcomes.

213. Which of the following outcomes is most likely to be associated with the Intergroup study that compared concurrent chemoradiation therapy administered twice daily versus thoracic radiation therapy administered once daily for small cell lung cancer?
A. The twice daily fractionation dose was 1.2 Gy with 4-hour intervals.
B. The 2-year and 5-year survival rates were 47% for concurrent chemoradiation BID and 26% for once daily radiation, respectively.
C. The total dose of daily thoracic radiation therapy was 54 Gy at 1.8 Gy per fraction.
D. There was no significant grade 3 esophagitis among patients who received chemoradiation therapy twice daily.

Correct answer is B. Reference: Turrisi, et. al. New England Journal of Medicine. 1999;340:265-71.

234. Which of the following mediastinal nodal stations can be biopsied using cervical mediastinoscopy?
A. Level 6 (paravascular/paraaortic)
B. Level 7 (anterior subcarinal)
C. Level 10 (hilar)
D. Level 12 (lobar)

Correct answer is B. RATIONALE: Mediastinal lymph node levels approachable through the cervical mediastinoscopy include level 2 (upper paratracheal) on either side, level 4 (lower paratracheal) on either side, and level 7 (subcarinal, or more precisely, the anterior aspect of those nodes). Level 5 (aortopulmonary) or level 6 (paravascular) lymph nodes can be biopsied through the Chamberlain procedure (paramedian sternotomy). Level 10 and 12 lymph nodes are within the pleural reflection, and therefore, they are not accessible to the mediastinoscopy.

271. Which of the following statements best reflects postoperative management and outcomes for NSCLC?
(A) Chemotherapy is recommended for after a complete resection for a T1bN0M0 disease.
(B) Adjuvant chemotherapy is recommended for a patient with T1bN0 NSCLC after complete resection.
(C) A randomized trial (CALGB) has demonstrated the need for adjuvant chemotherapyradiation for resected T2N01 disease.
(D) Local failure rate after wedge resection is less than 10 percent.

Key: C
Rationale: Choice A is incorrect because adjuvant chemotherapy is only recommended for T2N0 (stage IB) patient with >4 cm tumors, and stage II-III tumors. The CALGB randomized trial for T2N0 NSCLC failed to demonstrate the benefit of adjuvant chemotherapy, except for larger tumors >4cm in an unplanned subset analysis. The local failure rate after wedge resection is 18 percent. The local failure rate of 6% is after a lobectomy.

130. What is the biological effective dose (BED) when 48 Gy of SBRT is given in 3 fractions for stage I NSCLC?
(A) 72 Gy
(B) 84 Gy
(C) 100 Gy
(D) 132 Gy

Key: D
Rationale: The linear quadratic (LQ) equation has been the generally accepted method for comparing different RT fractionation schemes. Use of this equation yields a biologically effective dose (BED) for a given schedule against which the potency another can be compared. BED=nd(1+d/αβ radio) where dose, d, represents individual fraction size in Gy, n the number of individual doses of radiation, and α and β are intrinsic properties of the tumor cells. For comparison purposes, an α/β ratio of 10 is assumed for tumor killing effects and was used for calculations by the RTOG trials. 60 Gy in 30 fractions would have BED of 74 Gy, 70 Gy in 35 fractions equal BED of 84 Gy. 48 Gy in 3 fractions would equal BED of 132 Gy.

238. Patients who have small cell carcinoma of the lung are LEAST likely to have which of the following chromosomal abnormalities?
A. p53
B. del(3p)
C. Rb gene inactivation
D. K-ras gene

Correct answer is D. RATIONALE: The K-ras gene is seen in non-small cell lung cancer (NSCLC), but it is rarely seen in small cell lung cancer (SCLC). Gunderson and Tepper. Clinical Radiation Oncology. 2nd edition. 2007.

248. What is the overall incidence of grade 3 esophagitis in patients receiving concurrent chemotherapy and twice-daily radiation therapy for limited-stage small cell carcinoma of the lung?
A. 15%
B. 30%
C. 45%
D. 70%

Correct answer is B. Reference: Grade 3 esophagitis (defined as an inability to swallow solids, requiring narcotic analgesics or the use of a feeding tube) occurred in 27% of patients receiving concurrent chemotherapy and twice-daily radiation therapy for limited-stage small cell carcinoma of the lung. Reference: New England Journal of Medicine. 1999;(340):265.

261. The chemotherapy regimen most commonly used for treating limited-stage small cell carcinoma consists of:
A. cisplatin and etoposide.
B. cisplatin and irinotecan.
C. carboplatin and etoposide.
D. cyclophosphamide and vincristine.

Correct answer is A. RATIONALE: All of the combinations of chemotherapy are used, but cisplatin and etoposide are most commonly used for treating limited-stage small cell carcinoma.

270. Which of the following findings was demonstrated by the RTOG 98-01 phase III trial that studied the use of amifostine for prevention of treatment-related esophagitis?
A. A significantly higher number of lung cancer deaths were associated with the amifostine arm of the trial due to "tumor protection" by amifostine.
B. Significantly less high-grade esophagitis was associated with the control arm of the trial.
C. Significantly less high-grade esophagitis was associated with the amifostine arm of the trial.
D. There was no difference in the incidence of high-grade esophagitis between the two arms of the trial.

Correct answer is D. RATIONALE: The Radiation Therapy Oncology Group (RTOG) 98-01 trial was the largest phase III randomized trial studying the use of amifostine to prevent treatment related esophagitis during chemoradiotherapy for lung cancer. Unfortunately, no significant differences in the rates of treatment-related esophagitis were observed, and therefore, amifostine should not be used off study in that setting. However, no "tumor protection," and therefore, no survival decrement in the amifostine arm was observed either when compared to the control arm.

285. Thoracic radiation therapy with concurrent chemotherapy is planned for a 64-year-old man with stage IIIB non-small cell lung cancer. CT scan reveals a 30-cm separation at the isocenter. The most appropriate field arrangement and beam energy is:
A. AP-PA fields, 6-MV photon beam.
B. AP-PA fields, 18-MV photon beam.
C. multiple fields, 6-MV photon beam.
D. multiple fields, 18-MV photon beam.

Correct answer is C. RATIONALE: Lower photon energies (6 MV to 10 MV) are recommended for the treatment of intrathoracic malignancies. Use of higher energies may be associated with a relative underdosing of the air-tumor interface due to the buildup effect. In addition, higher photon energies have a much larger penumbra in the air than lower energies. If multiple fields are used rather than AP-PA fields, the risk of underdosing of the tumor surface is diminished, and the treatment plan may be better optimized.

154. According to the latest AJCC staging manual for lung cancer, which of the following would be classified as a T3 tumor?
(A) A tumor >5 cm but ≤7 cm in greatest dimension
(B) Separate tumor nodule(s) in the same lobe
(C) Involvement of the main bronchus, ≥ 2 cm distal to the carina
(D) Visceral pleura invasion

Key: B
Rationale: Pertinent changes to the lung staging system from the 6th to 7th edition include (1) more discrete stratification of tumors by size and partitioning of larger (>7 cm) tumors into T3, (2) downstaging of multiple nodules to T3 (same lobe), T4 (same lung), and M1a (contralateral lobe), and (3) upstaging of malignant pleural effusion from T4 to M1a.
References: AJCC Cancer Staging Manual, 7th edition, 2009.

305. Which of the following statements about the results of the phase III EORTC trial (08993) that randomized patients with extensive-stage small cell lung cancer to receive prophylactic cranial irradiation (PCI) or no PCI is true?
A. Treatment with PCI doubled the 1-year survival with significant toxicity.
B. Treatment with PCI decreased symptomatic brain metastases, but had no impact on survival.
C. Treatment with PCI reduced symptomatic brain metastases and improved survival.
D. Patients who received PCI were less likely to develop extracranial metastases.

Correct answer is C. RATIONALE: Prophylactic cranial irradiation (PCI) significantly reduced symptomatic brain metastases and significantly improved disease-free survival and overall survival in patients with extensive-stage small cell lung cancer. Survival doubled with mild acute toxicity. Reference: New England Journal of Medicine. 2007;(357), pp 664-72.

309. Which of the following statements about radiation-induced pulmonary fibrosis is true?
A. It is effectively treated with hyperbaric oxygen.
B. Its severity is reduced with concurrent chemotherapy.
C. It is managed conservatively and is usually irreversible.
D. It is managed with surgical excision.

Correct answer is C. RATIONALE: Radiation-induced pulmonary fibrosis is generally
irreversible; therefore, the emphasis has been on its prevention from the outset rather than its treatment after the fact. Hyperbaric oxygen has not been convincingly shown to be effective for the treatment of radiation-induced pulmonary fibrosis. Concurrent chemotherapy, especially with radiomimetic agents, usually increases rather than decreases the incidence of late effects. Surgical manipulation of irradiated tissues should be performed only by experienced cancer surgeons, but results are often unpredictable.

341. A 65-year-old woman presents with a cough and dyspnea on exertion. Thoracic CT scan reveals a 2.5-cm spiculated mass (adenocarcinoma) in the right upper lobe and no enlarged hilar or mediastinal lymph nodes. The patient has no metastatic disease, and results of mediastinoscopy are normal. Ipsilateral hilar lymph node involvement is noted intraoperatively. The most appropriate next step is:
A. right upper lobectomy only.
B. right upper lobectomy and mediastinal dissection.
C. concurrent chemoradiotherapy and no further surgery.
D. wedge resection of the right upper lobe tumor and ipsilateral mediastinal lymph node sampling.

Correct answer is B. RATIONALE: This patient has a T1N1M0 stage IIA non-small cell lung cancer (NSCLC). The standard surgical procedure for stages I and II NSCLC is lobectomy with mediastinal lymph node dissection, even in view of preceding normal mediastinoscopy. There is no reason to abandon surgery or perform the suboptimal wedge resection, since both approaches would compromise this patient’s outcome.

349. Which of the following statements about the use of radiation therapy for malignant pleural mesothelioma is true?
A. Mesothelioma does not respond favorably to radiation therapy.
B. Mesothelioma is more sensitive than small cell lung cancer to radiation therapy.
C. Mesothelioma is less sensitive than non-small cell lung cancer to radiation therapy.
D. Prophylactic radiation therapy effectively prevents tract seeding after thoracoscopy.

Correct answer is D. RATIONALE: Mesothelioma responds to radiation therapy, is less sensitive than small cell lung cancer, and is more sensitive than non-small cell lung cancer. Boutin, et al, randomized 40 patients with malignant pleural mesothelioma (MPMs) to prophylactic radiation therapy or no radiation after thoracoscopy. None of the 20 patients who received radiation therapy had tract seeding, compared with 8 (40%) of the control group. Reference: Chest. 1995;108:754-8.

353. Which of the following systemic syndromes is most commonly associated with thymoma?
A. Myasthenia gravis
B. Cushing syndrome
C. Hypercalcemia
D. Hypertension

Correct answer is A. RATIONALE: Myasthenia gravis is most frequently associated with thymoma. Other systemic syndromes associated with thymoma include hypogammaglobulinemia and RBC aplasia (10% to 15%), systemic lupus erythematosus (SLE), polymyositis, and myocarditis. Good syndrome is diagnosed in patients with a thymoma and combined B-cell/T-cell immunodeficiency. Hypercalcemia is more often associated with squamous cell lung cancer.

5. What approximate percentage of SCLC patients present with extensive disease in the U.S.?
(A) <20%
(B) 30-40%
(C) 60-70%
(D) >80

Key: C
Rationale: Approximately 35,000 Americans are diagnosed with small cell lung cancer annually. The incidence of extensive disease is 60-70%. This percentage of patients with ED has increased over the last 20 years, and this is at least partially due to stage migration secondary to routine use of CT scans, brain MRIs, and PET. PET alone upstages 8% of patients diagnosed with limited disease based on conventional staging (Bradley; J Clin Oncol. 22(16):3248-54, 2004; Niho; Lung Cancer. 57(3): 328-332007).

1. Hypofractionated stereotactic body radiation therapy (SBRT) of 60 Gy in 3 fractions is most appropriate for a patient who has medically inoperable NSCLC with a:
A. 3-cm tumor involving the chest wall.
B. 3-cm tumor involving the right mainstem bronchus.
C. 2-cm tumor involving the right lower lobe with hilar adenopathy.
D. 2-cm tumor involving the carina.

Correct answer is A. RATIONALE: The ineligibility for RTOG stereotactic body radiation therapy (SBRT) trial: Patients with T2 or T3 primary tumors of >5 cm or patients with T3 primary tumors involving the central chest and structures of the mediastinum. The primary tumor of any T-stage within or touching the zone of the proximal bronchial tree defined as a volume 2 cm in all directions around the proximal bronchial tree (carina, right and left main bronchi, right and left upper lobe bronchi, intermedius bronchus, right middle lobe bronchus, lingular bronchus, right and left lower lobe bronchi). A higher rate of airway stenosis was reported in a prior dose-escalating stud when 60 Gy was given in 3 fractions to tumors located in the central zone.

12. Which of the following two chemotherapy agents would be most appropriate to administer concurrently with thoracic radiation therapy for patients with limited-stage SCLC?
A. Cisplatin and etoposide
B. Cisplatin and irinotecan
C. Carboplatin and paclitaxel
D. Pemetrexed and carboplatin

Correct answer is A. RATIONALE: Cisplatin and etoposide remain the optimal regimen used with thoracic radiation therapy in limited-stage SCLC. Other agents tested or adding a third agent did not prove beneficial.

24. Which of the following statements about pure bronchioloalveolar carcinoma is true?
A. It is not associated with the expression of TTF-1 and CK7.
B. The EGFR gene mutation in patients with lung cancer is linked to bronchioloalveolar differentiation.
C. It requires the presence of lymphatic space invasion.
D. Stromal invasion is commonly associated.

Correct answer is B. RATIONALE: Pure bronchioloalveolar carcinoma (BAC) requires absence of invasion of stroma, pleura, and lymphatic spaces. Nonmucinous BAC expresses TTF-1, CK7, and lacks CK20.

226. Which of the following statements is TRUE regarding INT 0160 trial exploring chemoradiation followed by resection for patients with superior sulcus NSCLC?
(A) Most patients were unable to complete neo-adjuvant chemoradiation.
(B) Local failure was the most common site of disease progression.
(C) Pathologic complete response or minimal microscopic disease occurred in > 50% of patients undergoing resection.
(D) Patients with T3 tumors had double the overall survival of those with T4 tumors.

Key: C
Rationale: In this trial, a very select group of patients with superior sulcus NSCLC (T3-4, N0-1) underwent induction chemoradiation with cisplatin and etoposide concurrent with 45 Gy radiation. Five-year overall survival for the entire group was 44% and 54% for those who underwent a complete resection, which is nearly double that reported for those with N2 disease treated with definitive chemoradiotherapy or neoadjuvant chemoradiotherapy.
References: Rush et al, JCO 2007.

227. Which of the following statements BEST reflects the management of early stage non-small cell lung cancer (NSCLC)?
(A) The 2-year local control rate after 60 Gy of conventional radiotherapy is around 80%.
(B) The 3-year local control rate after SBRT to 54 Gy in 3 fractions is >95%.
(C) The 5-year survival rate after SBRT for medically inoperable stage I disease is around 70%.
(D) Adjuvant chemotherapy is indicated after completion of SBRT for larger tumors.

Key: B
Rationale: A is incorrect because conventionally fractionated RT to 60 Gy is around 30%. The 5 year survival after SABRBRT in inoperable patients is around 30-50%. Giving adjuvant chemotherapy after SABRBRT is not a standard, regardless of tumor size. The correct choice is B, which was demonstrated to be 98% after 3 years based on RTOG 0236 (Timmerman JAMA 2010).

53. What is the expected progression-free survival rate after hypofractionated stereotactic body radiation therapy (SBRT) for stage IA NSCLC?
A. 30%
B. 50%
C. 70%
D. 90%

Correct answer is D. RATIONALE: Several studies, some of which were prospective (nonrandomized), have shown the local control rate to be 90% or higher for stage IA NSCLC treated with hypofractionated SBRT. REFERENCE: Fowler JF, Tome WA, Welsh JS. Estimation of the required doses in stereotactic body radiation therapy. Kavanagh BD and Timmerman RD, eds. Stereotactic Body Radiation Therapy. Lippincott, Williams & Wilkins; 2005.

77. According to a recently published (Slotman) randomized trial, PCI improves survival in patients who have SCLC with:
A. limited-stage disease irrespective of response to chemotherapy.
B. extensive-stage disease irrespective of response to chemotherapy.
C. limited-stage disease with no progression during chemotherapy.
D. extensive-stage disease with no progression during chemotherapy.

Correct answer is D. RATIONALE: Based on the PCI meta-analysis, patients who had limited-stage SCLC and achieved a complete response to chemotherapy experienced improved survival rates with PCI. In the European randomized trial (Slotman, et al), patients who had extensive-stage small cell lung cancer and achieved any response to chemotherapy were eligible and experienced improved survival rates.

121. Postoperative radiation therapy may be indicated for a patient who has what stage of NSCLC and is receiving what type of therapy?
A. T1N1 disease and is receiving adjuvant chemotherapy
B. T1N1 disease and is unable to receive adjuvant chemotherapy
C. T2N0 disease and is receiving adjuvant chemotherapy
D. T2N0 disease and is unable to receive adjuvant chemotherapy

Correct answer is B. RATIONALE: The recently published update of the Adjuvant Navelbine International Trialist Association (ANITA) randomized trial demonstrated that postoperative radiation therapy provided a survival benefit in patients with both pN1 and pN2 NSCLC disease when these patients were unable to receive adjuvant chemotherapy. REFERENCE: International Journal of Radiation Oncology, Biology, Physics (Int J Radiat Oncol Biol Phys). 72(3):695–701.

126. Postoperative chemoradiation therapy is most likely to improve survival in patients with which of the following stages of NSCLC?
A. T1N1
B. T1N2
C. T2N1
D. T3N0

Correct answer is B. RATIONALE: A recent assessment of postoperative radiation therapy (RT) in 7,465 patients with resected stage II or III non-small cell lung cancer (NSCLC) found that postoperative RT increased survival in patients with N2 disease but not in those with N1 or N0 disease. Only chemotherapy, not chemoradiation, should be given for N1 disease based on currently available data. The recently published update of the Adjuvant Navelbine International Trialist Association (ANITA) randomized trial demonstrated that postoperative RT provided a survival benefit in patients with both pN1 and pN2 NSCLC disease when these patients were unable to receive adjuvant chemotherapy. REFERENCES: Lally, et al. Journal of Clinical Oncology. 2006;24(19):2998-3006. International Journal of Radiation Oncology, Biology, Physics (Int J Radiat Oncol Biol Phys). 72(3):695–701.

137. Which of the following AJCC staging criteria for NSCLC is correct?
A. Tumor invasion of the carina is T3.
B. Tumor invasion of the phrenic nerve is T4.
C. Involvement of the ipsilateral supraclavicular lymph node is N3.
D. Involvement of the contralateral mediastinal lymph node is N2.

Correct answer is C. REFERENCE: AJCC lung cancer staging. AJCC Cancer Staging Manual, 6th edition.

146. Which of the following treatment modalities should be recommended for a patient with a stage II, centrally located NSCLC that is medically inoperable?
A. Conventional fractionated radiation therapy (66 Gy in 33 fractions)
B. Stereotactic body radiation therapy (60 Gy in 3 fractions)
C. Radiofrequency ablation
D. Cryotherapy

Correct answer is A. RATIONALE: The central location of the cancer makes this patient a poor candidate for stereotactic body radiation therapy (SBRT), which would cause a higher rate of airway stenosis. Radiofrequency ablation has not been established as definitive therapy. Cryotherapy can be useful for palliation of endobronchial disease.

73. Which method is used for detection of the oncogenic EML4-ALK translocation in patients with lung adenocarcinoma?
(A) DNA sequencing
(B) Polymerase chain reaction
(C) Fluorescent in situ hybridization
(D) Immunofluorescence microscopy

Key: C
Rationale: Fluorescent in-situ hybridization (FISH) is used clinically to identify a chromosomal translocation resulting in an aberrant fusion gene that encodes a constitutively active ALK. It took only 3 years from discovery of this alteration in lung cancers to the report of dramatic clinical responses upon treatment with the small molecule inhibitor crizotinib, leading to FDA approval in 2011. Whether these alterations predominate in stage IV disease remains to be determined. This is an important example of molecular profiling of cancers impacting therapy.
References: Shaw et al. J Clin Oncol 2011; 27:4247-4253.

156. According to the Lung Intergroup 0139 trial, which of the following outcomes was demonstrated in the trimodality arm for patients with stage IIIA NSCLC?
A. There was a statistically significant improvement in disease-free and overall survival.
B. There was a statistically significant improvement in local control.
C. A complete resection was achieved in only 50% of patients.
D. The trimodality treatment approach offered the best survival rate when a pneumonectomy was incorporated.

Correct answer is B. RATIONALE: Seventy-one percent of patients had a complete resection. The local relapse rate after trimodality treatment was significantly improved (10% vs. 22%), and the disease-free survival rate also was significantly improved. There was a trend for increased 5-year overall survival associated with the trimodality approach; however, that is not optimal when a pneumonectomy is required due to the increase in mortality rates.

173. Which of the following agents is most effective when combined with cisplatin for treatment of mesothelioma?
A. Pemetrexed
B. Gemcitabine
C. Navelbine
D. Docetaxel

Correct answer is A. RATIONALE: In a randomized phase III trial, pemetrexed plus cisplatin was associated with improved survival compared with cisplatin alone for patients with malignant pleural mesothelioma (MPM). Median time to progression was significantly longer in the pemetrexed/cisplatin arm: 5.7 months versus 3.9 months (P =.001). Response rates were 41.3% in the pemetrexed/cisplatin arm versus 16.7% in the control arm (P <.0001). REFERENCE: Journal of Clinical Oncology. July 15, 2003;21(14):2636-44.

180. Based on the Masaoka staging system for thymoma, which of the following stages is associated with macroscopic invasion into adjacent organs?
A. IIA
B. IIB
C. III
D. IV

Correct answer is C. RATIONALE: This item refers to the Masaoka staging system. Macroscopic invasion into adjacent organs is classified as stage III disease.

194. Which of the following statements about pleural mesothelioma is true?
A. The epithelial subtype has the worst prognosis.
B. It does not spread to the mediastinal lymph nodes.
C. Thirty percent of cases are associated with a history of asbestos exposure.
D. Combined modality therapy results in an overall median survival of 14 to 16 months.

Correct answer is D. RATIONALE: Patients with mixed or sarcomatous cell disease or mediastinal lymph node involvement have a worse overall survival. Seventy percent to eighty percent of mesothelioma cases were associated with a history of asbestos exposure. Patients who receive combined modality treatment for mesothelioma have an overall median survival of about 14 to 16 months.

198. Approximately what percent of patients with NSCLC present with stage III disease?
A. 10%
B. 20%
C. 30%
D. 50%

Correct answer is C. RATIONALE: According to AJCC 2002, stage I = 25%, stage II = 7%, stage III = 32%, and stage IV = 36%. Therefore, the majority of patients will have advanced-stage lung cancer at diagnosis. REFERENCE: AJCC Cancer Staging Manual, 6th edition.

233. According to the Lung Intergroup 0139 trial, the mortality rate was highest after which of the following surgical procedures for stage III NSCLC?
A. Right-sided pneumonectomy
B. Left-sided pneumonectomy
C. Left lower lobectomy
D. Right upper lobectomy

Correct answer is A. RATIONALE: The mortality rate in patients receiving trimodality treatment was 7.9%, and the mortality rate in patients receiving chemoradiation was 2.1%. Complex pneumonectomy had a much higher mortality rate than that of simple pneumonectomy. The mortality rate after right-sided complex pneumonectomy was 50%; the mortality rate after left-sided complex pneumonectomy was 16%. The mortality rate for lobectomy was 1%, which increased to 26% after pneumonectomy.

242. Which of the following thoracic radiation therapy regimens administered concurrently with chemotherapy improves overall survival for limited-stage SCLC?
A. 45 Gy in twice-daily fractions of 1.5 Gy each
B. 48 Gy in twice-daily fractions of 1.5 Gy each, with a 2-week break after 24 Gy
C. 61.2 Gy in once-daily fractions of 1.8 Gy each for 5 weeks and twice-daily fractions of 1.8 Gy each for the last 9 days
D. 70 Gy in once-daily fractions of 2 Gy each

Correct answer is A. RATIONALE: The radiation therapy regimen of 45 Gy bid is the only one showing a survival benefit in a randomized Phase III trial. REFERENCE: Turrisi. Intergroup trial. New England Journal of Medicine (NEJM). January 28, 1999;340(4):265-71.

263. What is the expected rate of CNS metastasis after systemic chemotherapy and PCI for patients with extensive-stage SCLC?
A. 05%
B. 15%
C. 30%
D. 50%

Correct answer is B. RATIONALE: The incidence of brain metastases in patients who had extensive-stage SCLC and received PCI was 15%. The incidence of brain metastases in patients who had limited-stage SCLC and received PCI was approximately 20%.

270. According to the 1999 Intergroup (Turrisi) trial, accelerated hyperfractionated thoracic radiation therapy beginning with the first cycle of chemotherapy significantly improved 5-year survival as compared with concurrent once-daily radiation therapy by:
A. 30%.
B. 20%.
C. 10%.
D. 05%.

Correct answer is C. RATIONALE: The survival rates for patients receiving once-daily radiation therapy were 41% at 2 years and 16% at 5 years. REFERENCE: New England Journal of Medicine. Jan 28, 1999;340(4):265-71.

251. Which of the following statements is TRUE regarding lung cancer screening?
(A) Screening annual CXR with sputum cytology improves lung cancer detection.
(B) The Cochrane Meta-analysis (2004) reports an improvement in all-cause mortality with lung cancer screening.
(C) The International Early Lung Cancer Action Project (I-ELCAP) is a randomized trial to validate screening CT.
(D) Screening with low dose CT improves overall survival as compared to CXR.

Key: D
Rationale: All the statements are false except for D, which is the summary of the NLST Research Group published in the NEJM in 2011. Low dose CT improves all cause mortality by 6.7% compared to CXR alone in high risk patients. A is incorrect because CXR + sputum cytology did not improve cancer specific survival. The meta-analysis demonstrated no benefit of screening. The I-ELCAP is an observational study of 31,567 pts with screening CTs.

239. The EORTC randomized clinical study comparing PCI with observation in patients with extensive stage SCLC with any response to systemic chemotherapy showed PCI did all the following, EXCEPT:
(A) higher rate of CNS toxicity.
(B) the one-year cumulative risk of brain metastases was reduced from 40% to 15%.
(C) prolonged the disease-free and overall survival.
(D) reduced the risk of symptomatic brain metastases.

Key: A
Rationale: The cumulative risk of brain metastases within 1 year was 15% in the irradiation group and 40% in the control arm. The 1-year survival rate was 27% in the irradiation group and 13% in the control group. Prophylactic cranial irradiation reduced the incidence of symptomatic brain metastases and prolongs disease-free and overall survival.
References: Slotman B, Faivre-Finn C, Kramer G, et al. Prophylactic cranial irradiation in extensive small-cell lung cancer. N Engl J Med 2007; 357:664-672.

277. Which of the following toxicities is most common in patients who are receiving concurrent chemoradiation for NSCLC?
A. Pneumonitis
B. Esophagitis
C. Pericarditis
D. Myelitis

Correct answer is B. RATIONALE: Esophagitis can be observed in 40% to 60% of patients receiving concurrent chemoradiation therapy. Pneumonitis is much less common. Pericarditis and myelitis are rare in the modern era of radiation therapy.

283. Which of the following statements about the management of thymoma is true?
A. Chemotherapy is not effective.
B. A complete thymectomy is preferred for resectable disease.
C. Adjuvant radiation therapy is indicated for patients with Masaoka stage I disease.
D. Preoperative radiation therapy is indicated for patients with Masaoka stage I disease.

Correct answer is B. RATIONALE: For resectable disease, a complete thymectomy is preferred because recurrences have been observed in the residual thymus after a partial thymectomy. Several reports showed that 17% to 28% of patients who have thymomas develop a second malignancy after thymectomy. Thus, close follow-up is needed given that secondary malignancy is common in patients with thymomas.

299. The addition of early thoracic radiation therapy to chemotherapy for patients with limited-stage SCLC is expected to improve overall survival at 3 years by:
A. 00%.
B. 05%.
C. 10%.
D. 15%.

Correct answer is B. RATIONALE: In the meta-analysis of the thoracic radiation therapy trials, the overall survival rate was improved by 5.4% at 3 years in patients who received radiation therapy in addition to chemotherapy for limited-stage SCLC.

306. Which of the following outcomes is associated with cisplatin-based adjuvant chemotherapy for NSCLC?
A. Reduction in the risk of death by about 10%
B. Improved 5-year disease-free survival by about 10%
C. Absolute benefit of 10% in 5-year disease-free survival
D. Absolute benefit of 10% in 5-year overall survival

Correct answer is A. RATIONALE: According to the Lung Adjuvant Cisplatin Evaluation (LACE) and meta-analysis, cisplatin-based adjuvant chemotherapy improves survival among patients who have completely resected NSCLC with approximately 10% reduction in the risk of death, with absolute benefits in 5-year disease-free and overall survival rates of 5%. REFERENCES: Journal of Clinical Oncology (JCO). 2008;(26)3552-9. New England Journal of Medicine (NEJM). 2004;350(4):315-60.

263. The randomized clinical trial comparing thoracic radiation of 45 Gy given in once-daily to twice-daily in addition to current chemotherapy for limited-stage SCLC demonstrated that twice daily had significantly increased:
(A) Grade 4 esophagitis.
(B) Grade 4 myelotoxicity.
(C) Grade 3 esophagitis.
(D) Grade 3 pneumonitis.

Key: C
Rationale: Grade 3 esophagitis was significantly more frequent with twice-daily thoracic radiotherapy, occurring in 27% of patients, as compared with 11% in once-daily group. There was no difference between groups in the incidence of grade 4 esophagitis.
References: Turrisi AT, Kim K, Blum R, et al. Twice-daily compared with once-daily thoracic radiotherapy in limited small-cell lung cancer treated concurrently with cisplatin and etoposide. N Engl J Med 1999; 340:265-271.

105. What would be the cure rate for patients with stage III NSCLC treated with either surgery or radiation therapy alone?
(A) <10%
(B) 20%
(C) 30%
(D) 50%

Key: A
Rationale: About 150,000 new cases of NSCLC are diagnosed every year in the United States. At least 40% will present with Stage III NSCLC. Surgical resection or radiotherapy alone will result in cure in less than 10% of patients (Mountain CF. Revisions in the international staging system for staging lung cancer. Chest 111:1710-1717, 1997).

187. What is the strongest predictor of survival for patients with IIIA NSCLC treated with neoadjuvant chemoradiation and resection?
(A) Total dose of radiation
(B) Intensity of chemotherapy
(C) Primary tumor response
(D) Pathologic nodal clearance

Key: D
Rationale: The strongest predictor of long- term survival was mediastinal nodal clearance, which had a hazard ratio of 0.22, (p=.0003) (Betticher et al., J Clin Oncol. 21(9): 1752-1759, 2003). Most importantly, in the phase III trial, INT 0139, nodal clearance predicted outcome. Patients with N0 resection specimens demonstrated a 41% 5-year survival versus 24% for those with pN1-3 in the surgical specimen (Albain et al. J Clin Onc. 23:16S: 7014, 2005). Clearly, studies of induction therapy (chemotherapy or chemoradiotherapy) followed by surgical resection have consistently demonstrated that mediastinal nodal sterilization is a powerful predictor of outcome and can serve as a surrogate marker.


317. Which of the following paraneoplastic syndromes are associated with SCLC?
A. SIADH, hypercalcemia, and Cushing syndrome
B. SIADH, limbic encephalitis, and Lambert-Eaton syndrome
C. Lambert-Eaton syndrome and hypercalcemia
D. Cushing syndrome and thrombocytosis

Correct answer is B. RATIONALE: Hypercalcemia is classically encountered in squamous cell carcinoma, but not in SCLC. Thrombocytosis has not been described, although thrombocytopenia may occur.

7. What is the most appropriate treatment strategy for a patient who underwent a
lobectomy and mediastinal lymph node dissection with final pathology revealing a stage
T2N0 SCLC?
A. Close surveillance
B. Thoracic radiation therapy only
C. Thoracic radiation therapy, chemotherapy, and PCI
D. Chemotherapy and PCI only

Correct answer is D. RATIONALE: Stage I small cell lung cancer (SCLC) is diagnosed in less than 5% of patients with SCLC. Patients who undergo complete resection (preferably by a lobectomy with either mediastinal nodal dissection or sampling) should receive postoperative chemotherapy, followed by PCI. REFERENCE: NCCN Practice Guidelines in Oncology. 2010.

22. Which of the following characteristics is associated with patients who have SCLC?
A. Approximately 10% of patients are nonsmokers.
B. Approximately 10% of patients present with a superior sulcus tumor.
C. Approximately 30% of patients present with limited disease.
D. It accounts for 30% of all lung cancers.

Correct answer is C. RATIONALE: Small cell lung cancer (SCLC) accounts for 15% of all lung cancers. Nearly all cases of SCLC are attributed to cigarette smoking. About one third of patients present with limited disease confined to the chest. Less than 2% of patients with SCLC present with a superior sulcus tumor.

36. What percent of patients diagnosed with SCLC present with superior vena caval obstruction?
A. <5%
B. 10%
C. 20%
D. 30%

Correct answer is B. RATIONALE: Superior vena caval obstruction is present at diagnosis in 10% of patients with small cell lung cancer (SCLC). REFERENCE: Sculier JP, et al. Superior vena caval obstruction syndrome in small cell lung cancer. Cancer. 1986;57:847.

51. According to a prospective randomized study for patients who had limited-stage SCLC and achieved a complete response to therapy, which of the following results was associated with high-dose (36 Gy) versus standard-dose (25 Gy) PCI?
A. Increased 2-year disease-free survival rate
B. Increased 2-year overall survival rate
C. Decreased isolated CNS relapse rate
D. Decreased treatment compliance rate

Correct answer is C. RATIONALE: The corresponding 2-year incidence rates of total brain metastases in patients with small cell lung cancer (SCLC) are 29% (95% CI 24–35) in the standard radiation dose group and 23% (18–29) in the higher radiation dose group. Comparison of the cumulative incidence of total brain metastases curves by adjusted Gray’s test yields an HR of 0.76 (0.54–1.05, p=0.10), with a 2-year difference of 6% (22% vs 16%) favoring the higher radiation dose group. When brain metastasis is considered as an isolated first site of failure, the HR is 0.48 (0.29–0.81, p=0.005), and the 2-year difference is also 6% (12% vs 6%) favoring the higher radiation dose group. REFERENCE: Lancet Oncology. 2009;10:467–74.

66. Thoracic radiation therapy improves the absolute local control rate in patients with limited-stage SCLC by:
A. 10%.
B. 15%.
C. 20%.
D. 25%.

Correct answer is D. RATIONALE: Thoracic radiation therapy improves local control rates by 25% in patients with limited-stage small cell lung cancer (SCLC) and is associated with improved survival. REFERENCES: Pignon, et al. New England Journal of Medicine (NEJM). 1992;327:1618-1624. Payne, et al. Journal of Clinical Oncology. 1992;10:890-896.

82. The randomized trial by EORTC and RTOG comparing standard PCI dose to higher-dose in patients with limited-stage SCLC showed the higher dose of PCI:
(A) improved CNS disease-free survival.
(B) reduced overall mortality rates.
(C) did not reduce brain metastases.
(D) increased the rate of CNS toxicity.

Key: C
Rationale: No significant reduction in the total incidence of brain metastases was observed after higher radiotherapy dose for PCI when compared with standard radiotherapy dose for PCI, but there was a significant increase in mortality. PCI at 25 Gy should remain the standard of care in limited-stage SCLC.
References: LePechoux C, Dunant A, Senan S, et al. Standard-dose versus higher-dose prophylactic cranial irradiation (PCI) in patients with limited-stage small-cell lung cancer in complete remission after chemotherapy and thoracic radiotherapy (PCI 99-01, EORTC 22003-08004, RTOG 0212, and IFCT 99-01): a randomized clinical trial. The Lancet 2009; 10(5):467-474.

88. According to the ANITA trial, which of the following stages of NSCLC is most likely to
have the best response to postoperative chemotherapy and radiation therapy?
A. pT4N1M0
B. pT3N0M0
C. pT2N1M0
D. pT1N2M0

Correct answer is D. RATIONALE: The ANITA randomized trial showed postoperative
radiation therapy (RT) had a negative effect on pN1 non-small cell lung cancer (NSCLC)
disease when postoperative chemotherapy was given. Five-year survival was 40% vs. 56% with
or without postoperative RT. However, postoperative RT had a positive effect on pN2 disease
even when postoperative chemotherapy was given. Five-year survival was 47.4% vs. 34% with
or without postoperative RT. REFERENCE: International Journal of Radiation Oncology,
Biology, Physics. 2008;72:690-701.

103. Based on a retrospective analysis of the ANITA trial, which of the following stages of
NSCLC is most likely to have the best response to postoperative radiation therapy
alone?
A. pT3N0M0
B. pT2N1M1
C. pT2N1M0
D. pT2N0M0

Correct answer is C. RATIONALE: The ANITA randomized trial showed postoperative
radiation therapy (RT) had a negative effect on pN1 non-small cell lung cancer (NSCLC)
disease when postoperative chemotherapy was given, but it may have a positive effect when
RT alone is used. Five-year survival is 42.6% vs 31.4% with or without postoperative RT. LCSG
773 also showed better local control in patients who received postoperative RT for pN1 NSCLC
disease; the local recurrence rate was 3% vs 41% with or without RT. REFERENCE:
International Journal of Radiation Oncology, Biology, Physics (Int J Radiat Oncol Biol Phys).
2008;72:690-701.

246. Which of the following statements is TRUE about pleural mesothelioma?
(A) Only about 30% of all cases are associated with asbestos exposure.
(B) Sarcomatoid histology is the most common subtype.
(C) They rarely metastasize to lymph nodes.
(D) Epithelioid subtype has better prognosis than biphasic subtype.

Key: D
Rationale: The majority of pleural mesothelioma patients have history of asbestos exposure. There are three main histologic subtypes of mesothelioma: epithelioid, biphasic, and sarcomatoid. Epithelioid tumors are most common and have a better prognosis than biphasic and sarcomatoid tumors. Nodal metastasis is not uncommon.

133. Compared to sequential chemoradiation therapy, concurrent chemoradiation therapy used for patients with stage III NSCLC is more likely to:
A. decrease local recurrence.
B. decrease overall survival rates.
C. increase the risk of distant metastasis.
D. increase overall survival rates.

Correct answer is D. RATIONALE: Concurrent chemoradiotherapy has been shown to improve survival in patients with stage III non-small cell lung cancer (NSCLC), but the toxicity associated with the treatment is significant. Therefore, it should be considered only for a patient with a good-performance status. Even with concurrent chemoradiotherapy, local control is still not optimal, and dose escalation is being explored.

149. Which of the following statements about the 2009 AJCC TNM staging criteria for lung cancer is true?
A. Tumor size of >5 cm has been reclassified from stage T2 to stage T3.
B. Multiple tumor nodules in the same lobe have been reclassified from stage T4 to
stage T3.
C. The N stage classification has been changed to reflect the number of lymph nodes
involved.
D. The M stage classification has not changed.

Correct answer is B. RATIONALE: This item tests the summary changes of the 2009 AJCC
lung cancer staging classifications: The T-stage classifications have been redefined: Stage T1
has been subclassified into stage T1a (≤2 cm in size) and stage T1b (2 cm to 3 cm in size).
Stage T2 has been subclassified into stage T2a (3 cm to 5 cm in size) and stage T2b (5 cm to 7
cm in size). Stage T2 (>7 cm in size) has been reclassified as stage T3. Multiple tumor nodules
in the same lobe have been reclassified from stage T4 to stage T3. Multiple tumor nodules in
the same lung but a different lobe have been reclassified from stage M1 to stage T4. The N
stage classifications have remained unchanged. The M stage classifications have been
redefined. Stage M1 has been subdivided into stage M1a and stage M1b. Malignant pleural
and pericardial effusions have been reclassified from stage T4 to stage M1a. Separate tumor
nodules in the contralateral lung are considered stage M1a. Stage M1b designates distant
metastases.

156. Which of the following statements about the use of a FDG-PET scan for patients with NSCLC is true?
A. It is useful in measuring the exact tumor size.
B. It improves the accuracy of lymph node staging by 10% to 20% compared to a CTscan.
C. It has a high specificity and low sensitivity in detecting lymph node disease.
D. It is not as sensitive as a bone scan in detecting metastasis of the bone.

Correct answer is B. RATIONALE: A PET scan is very sensitive in detecting a high metabolic
state, such as in tumors, but provides poorer spatial resolution in cross-sectional anatomy as
compared to CT or MRI scans. A PET scan has been shown to be as sensitive as a bone scan
in detecting bone metastasis.

164. Which of the following treatments would be most appropriate for a patient with poor pulmonary function and stage I NSCLC involving the right hilum?
A. Radiofrequency ablation
B. Conventional fractionated radiation therapy
C. Stereotactic body radiation therapy
D. Wedge resection, followed by brachytherapy

Correct answer is B. RATIONALE: The 2009 Practice Guidelines by the National
Comprehensive Cancer Network (NCCN) recommends conventional fractionated radiation
therapy for medically inoperable early-stage non-small cell lung cancer (NSCLC). Hilar
involvement precludes stereotactic body radiation therapy.

168. What is the recommended V20 value for patients who receive IMRT after extrapleural
pneumonectomy for mesothelioma?
A. <7%
B. 17%
C. 23%
D. 30%

Correct answer is A. RATIONALE: The standard recommendation of keeping V20 below 35%
for patients who have lung cancer (when both lungs are counted as the total lung volume) and
are receiving thoracic radiation therapy (RT) does not apply to patients with mesothelioma
following pneumonectomy. The remaining lung has to be protected as much as possible from
RT, and several institutions (Beth Israel/Harvard; Duke; MD Anderson) described severe
pulmonary toxicity if the V20 was not kept below 7%. In addition, V5 is emerging as an
important parameter to watch, since IMRT frequently “floods” the remaining lung with low-dose
RT.

179. Which of the following treatment strategies is most appropriate for a patient who has
stage IIIB NSCLC with KPS of >70?
A. Concurrent chemotherapy and fractionated radiation therapy
B. Neoadjuvant chemotherapy, followed by concurrent chemoradiation
C. Conventional fractionated radiation therapy, followed by chemotherapy
D. Oligo fractionated stereotactic radiation therapy, followed by chemotherapy

Correct answer is A. RATIONALE: The standard treatment for a patient with stage IIIB nonsmall
cell lung cancer (NSCLC) is concurrent chemoradiation with fractionated radiation
therapy. This is supported by at least three randomized phase III trials: The West Japanese
study, Czech Republic study, and RTOG 9410. The CALGB 39801 study did not show a
significant advantage of survival in patients treated with neoadjuvant carboplatin and Taxol
before concurrent chemoradiation. The HOG Phase III study failed to show a significant benefit
of adjuvant docetaxel (Taxotere) chemotherapy, while the Locally Advanced Multimodality
Project (LAMP) did report longer median survival in patients treated with fractionated radiation
therapy with concurrent and adjuvant carboplatin and Taxol. REFERENCES: Zatloukal, et al.
Concurrent versus sequential chemoradiotherapy with cisplatin and vinorelbine in locally
advanced non-small cell lung cancer: a randomized study. Lung Cancer. Oct 2004;46(1):87-98.
Furuse, et al. Phase III study of concurrent versus sequential thoracic radiotherapy in
combination with mitomycin, vindesine, and cisplatin in unresectable stage III non-small cell
lung cancer (NSCLC). Journal of Clinical Oncology (J Clin Oncol). Sep 1999;17(9):2692-9.
RTOG 9410 is to be published soon. Fournel, et al. Randomized phase III trial of sequential
chemoradiotherapy compared with concurrent chemoradiotherapy in locally advanced non-small
cell lung cancer: Groupe Lyon-Saint-Etienne d'Oncologie Thoracique-Groupe Français de
Pneumo-Cancérologie NPC 95-01 Study. Journal of Clinical Oncology (J Clin Oncol). Sep 2005
1;23(25):5910-7. Hanna, et al. Phase III study of cisplatin, etoposide, and concurrent chest
radiation with or without consolidation docetaxel in patients with inoperable stage III non-small
cell lung cancer: the Hoosier Oncology Group and U.S. Oncology. Journal of Clinical Oncology
(J Clin Oncol). Dec 10, 2008;26(35):5755-60. Belani, et al. Combined chemoradiotherapy
regimens of paclitaxel and carboplatin for locally advanced non-small cell lung cancer: a
randomized phase II locally advanced multi-modality protocol. Journal of Clinical Oncology (J
Clin Oncol). Sep 1, 2005;23(25):5883-91. Epub 2005 Aug 8. Erratum in: Journal of Clinical
Oncology (J Clin Oncol). Apr 20, 2006;24(12):1966.

187. Which of the following conditions is a contraindication for extrapleural pneumonectomy
in patients with mesothelioma?
A. Pericardial pleural involvement
B. Presence of multiple ipsilateral pleural implants
C. Tumor extension through the diaphragm
D. Epithelial histologic subtype

Correct answer is C. RATIONALE: Practically all cases of mesothelioma will have multiple
pleural implants along the chest wall, all of which will be removed during pleurectomy and as
such would not constitute a contraindication to surgery. Pericardial pleura can be resected.
Extension through the diaphragm is a classic contraindication to complete resection. Epithelial
histologic subtype has a better prognosis than the sarcomatoid subtype and is more likely to
have prolonged survival after surgery.

194. Which of the following adverse effects most commonly occurs in a patient with lung
cancer after administration of concurrent chemoradiation?
A. Pneumonitis in 2 to 4 weeks during radiation treatment
B. Esophagitis in 3 to 4 weeks during radiation treatment
C. Pericarditis in 1 to 2 months after radiation treatment
D. Myelitis in 5 to 6 months after radiation treatment

Correct answer is B. RATIONALE: Esophagitis is the most common side effect (about 40%-
50%) during treatment in patients treated with concurrent chemoradiation, according to all of the
phase III trials of patients treated with concurrent chemoradiation therapy. Esophagitis often
occurs at 3-4 weeks from the commencement of radiation therapy.

119. Which of the following is NOT an adverse prognostic factor for patients with SCLC?
(A) Continuation of smoking
(B) Lower performance score
(C) Elevated alkaline phosphatase level
(D) Female gender

Key: D
Rationale: Patient factors influencing outcome are performance status and sex”. “Continuation of smoking will adversely affect the outcome”. “Other prognostic factors are elevated lactic dehydrogenase and alkaline phosphatase”.
References: Perez and Brady’s Principles and Practice of Radiation Oncology Fifth Edition, 2009 Wolters Kluwer/ Lippincott Williams & Wilkins; page 1100.

235. Which of the following features of pleural mesothelioma would indicate unresectable disease?
(A) Involvement of the endothoracic fascia
(B) Direct transdiaphragmatic extension to the peritoneum
(C) Extension into the mediastinal fat
(D) Nontransmural involvement of the pericardium

Key: B
Rationale: Only B is considered as T4 disease, which is locally advanced technically unresectable tumor. The rest features are T3 tumors and potentially resectable.

236. The radiation target volumes used in the randomized clinical trial (Turrisi, et. al.) comparing thoracic radiation of 45 Gy given in once-daily to twice-daily in addition to current chemotherapy for limited-stage SCLC included the gross tumor, bilateral mediastinal and:
(A) ipsilateral hilar lymph nodes.
(B) bilateral hilar lymph nodes.
(C) ipsilateral hilar and supraclavicular lymph nodes.
(D) bilateral hilar and supraclavicular lymph nodes.

Key: A
Rationale: The target volumes for thoracic radiotherapy (45 Gy, given either once or twice daily) beginning with cycle 1 of the chemotherapy (a total of four cycles of ciplatin plus etoposide was delivered) for limited-stage small-cell lung cancer included the gross tumor (as defined by chest CT scan), and the bilateral mediastinal and ipsilateral hilar lymph nodes. Irradiation of the uninvolved supraclavicular fossae was forbidden.
References: Turrisi AT, Kim K, Blum R, et al. Twice-daily compared with once-daily thoracic radiotherapy in limited small-cell lung cancer treated concurrently with cisplatin and etoposide. N Engl J Med 1999; 340:265-271.

118. Which of the following is TRUE regarding the INT 0139, a trial comparing neoadjuvant chemoradiation and surgical resection to definitive chemoradiation for locally advanced NSCLC?
(A) The majority of the treatment-related deaths occurred in patients undergoing pneumonectomy.
(B) The majority of the treatment-related deaths in both groups occurred during chemoradiation.
(C) Both the lobectomy and pneumonectomy after the chemoradiation provided identical overall survival.
(D) Chemotherapy for both groups consisted of carboplatin and taxol.

Key: A
Rationale: In this pivotal trial, only patients with stage IIIA (N2) NSCLC were eligible. Progression-free survival was superior for neoadjuvant chemoradiotherapy (median 12.8 vs. 10.5 mo, p=0.017), though overall survival was not. In an unplanned analysis, overall survival was improved for neoadjuvant chemoradiotherapy for patients who underwent lobectomy but not pneumonectomy. High rates of treatment related deaths occurred in patients undergoing pneumonectomy.
References: Albain et al, Lancet 2009.

201. According to the TNM mesothelioma staging system, a stage T3 tumor involves the:
A. ipsilateral visceral pleura.
B. ipsilateral parietal pleura.
C. chest wall in one focal area.
D. vertebral body.

Correct answer is C. RATIONALE: Mesothelioma evolves from the parietal pleura; therefore,
involvement of either parietal or visceral pleura denotes an early-stage tumor. A focal (in
contrast to diffuse) invasion of the chest wall is still potentially resectable; therefore, it is a stage
T3 tumor. Spinal invasion is not resectable and is a stage T4 tumor.

209. Which of the following factors is typically required when stereotactic body radiation
therapy is used to treat lung tumors?
A. Peripheral tumor location
B. Negative mediastinoscopic results
C. Photon energy of >15 MV
D. CTV expansion

Correct answer is A. RATIONALE: While 20% of patients with clinical early-stage lung cancer
may have involved lymph nodes, patterns of failure from stereotactic body radiation therapy
(SBRT) series do not demonstrate a high lymph node failure rate. Currently, CTV expansions
are NOT allowed on the ongoing RTOG 0618 study. Also, a photon energy of >15 MV is
specifically discouraged due to the typically small field used and lack of electronic equilibrium
from highly energetic laterally scattered electrons. Generally, peripheral tumor locations
(defined as tumors that are >2 cm from the bronchotracheal tree) are favored for SBRT, due to
the higher toxicity associated with the treatment of central lung lesions.

213. What is the most appropriate dose range of radiation therapy for a patient with a
completely resected Masaoka stage III thymoma?
A. 36 to 40 Gy
B. 41 to 45 Gy
C. 50 to 55 Gy
D. 60 to 66 Gy

Correct answer is C. RATIONALE: While the optimal postoperative radiation therapy (RT)
dose for patients with a resected thymoma is not fully established, many authors recommend a
dose of at least 50 Gy. Since local control rates achieved with doses between 50 to 54 Gy are
high (80-97%), it seems unnecessary to administer radiation doses of ≥60 Gy.

230. Which of the following treatment approaches is most appropriate for a 75-year-old man
who underwent a resection for thymic carcinoma with a focal residual tumor at the
mediastinal margin, which was marked with surgical clips?
A. 54 Gy to the entire tumor bed only
B. 60 Gy to the area marked with the clips only, with concurrent chemotherapy
C. 60 Gy to the entire tumor bed, with concurrent chemotherapy
D. 60 Gy to the entire tumor bed after chemotherapy

Correct answer is C. RATIONALE: Thymic carcinomas are highly malignant tumors with a high
risk of both local and distant failure despite aggressive therapy. Although no universally
accepted standard treatment exists, best results seem to be reported with a maximum safe
resection, followed by immediate postoperative radiation therapy (RT) given concurrently with
cisplatin-based combination chemotherapy. Due to the tumor’s infiltrative nature, most studies
use RT given to the entire tumor bed and mediastinum rather than to the area of suspected
residual disease only. Radiation doses in the range of 60 to 70 Gy may be associated with a
better outcome.

240. Which of the following groups of paraneoplastic syndromes is associated with thymoma?
A. Myasthenia gravis, hypocalcemia, and a monoclonal gamma-globulin spike in serum
level
B. Myasthenia gravis, hyponatremia, and a monoclonal gamma-globulin spike in serum
level
C. Myasthenia gravis, erythrocytosis, and hypogammaglobulinemia
D. Myasthenia gravis, red cell aplasia, and hypogammaglobulinemia

Correct answer is D. RATIONALE: Paraneoplastic syndromes listed in option D have been
associated with thymoma.

273. According to the Lung Cancer Study Group randomized trial, which of the following
results was demonstrated in patients who had a limited resection versus a lobectomy for
stage T1N0M0 NSCLC?
A. 75% increase in local recurrence
B. 75% increase in cancer-specific mortality
C. 30% increase in distant metastasis
D. 20% decrease in overall survival

Correct answer is A. RATIONALE: There was a 75% increase in local recurrence rate (p=0.02)
in patients with stage T1N0M0 non-small cell lung cancer (NSCLC), 30% increase in overall
mortality (p=0.08), and 50% increase in cancer-specific mortality (p=0.09). REFERENCE: Lung
Cancer Study Group. Annals of Thoracic Surgery (Ann Thorac Surg). 1995;60:615-22.

11. What is the treatment of choice for an otherwise healthy 55-year-old man who has
thymoma with pericardial involvement?
A. Surgical resection
B. Primary radiation therapy
C. Radiation therapy, followed by surgery
D. Chemoradiation, followed by surgery

Correct answer is A. RATIONALE: Surgical resection is the treatment of choice for a thymoma.
The goal of surgery is to completely excise the thymoma via total thymectomy with complete
resection of contiguous and noncontiguous disease, which may require resecting adjacent
structures such as the pericardium, phrenic nerve, pleura, lung, and even major vascular
structures. Pericardial involvement is considered to be a resectable disease.

31. What is the 5-year survival rate for patients with stage IV thymoma, based on the
Masaoka staging system?
A. 10%
B. 30%
C. 50%
D. 70%

Both options B and C were scored correct. RATIONALE: The Masaoka staging system is
useful for management and determination of prognoses for patients with thymoma. Patients with
stage I to stage III thymomas have a 5-year survival rate of about 70%. Patients with stage IV
disease have a 5-year survival rate of about 50%. REFERENCE: Park HS, et al. Thymoma. a
retrospective study of 87 cases. Cancer. 1994;73:2491-98.

38. Which of the following tumors is most likely to be found in the anterior mediastinum?
A. Small cell carcinoma
B. Adenoid cystic carcinoma
C. Paraganglioma
D. Germ cell

Correct answer is D. RATIONALE: Small cell carcinoma more commonly involves the middle
and posterior mediastinum; adenoid cystic carcinoma involves the trachea. Neurogenic tumors
are located in the posterior mediastinum. Thyroid tumors, thymoma, and lymphoma are the
most common tumors in the anterior mediastinum. Although mediastinal germ cell tumors are
rare, they are mostly located in the anterior mediastinum.

63. Which of the following mediastinal neoplasms is most likely to present with myasthenia
gravis?
A. Parathyroid adenoma
B. Small cell carcinoma
C. Teratoma
D. Thymoma

Correct answer is D. RATIONALE: Symptomatic paraneoplastic syndromes are generally
uncommon. However, the Lambert–Eaton myasthenic syndrome affects about 3% of patients
with small cell lung cancer; myasthenia gravis affects about 15% to 30% of patients with
thymoma. REFERENCE: Levy Y, et al. Malignant thymoma associated with autoimmune
diseases: a retrospective study and review of the literature. Seminars in Arthritis and
Rheumatism (Semin Arthritis Rheum). 1998;28:73-79.

79. A 45-year-old patient undergoes complete resection of a 5 cm, completely encapsulated,
type B2 thymoma. According to the SEER analysis, postoperative radiation therapy is:
A. indicated due to the tumor size.
B. indicated due to the B2 disease.
C. indicated with adjuvant chemotherapy.
D. not indicated.

Correct answer is D. RATIONALE: The SEER analysis showed clearly no benefit for the use of
postoperative radiation therapy (PORT) in patients with stage I (encapsulated) completely
resected thymoma. No conclusions were drawn based on tumor size or histology.
REFERENCE: Forquer JA, et al. International Journal of Radiation Oncology, Biology, Physics
(Int J Radiat Oncol Biol Phys). 2010.

83. A 55-year-old patient with stage T1bN2M0 adenocarcinoma of the right lung has
received preoperative chemotherapy and 45 Gy of thoracic radiation therapy. The
restaging CT scan of the chest after completion of chemoradiation shows progressive
disease in both the primary tumor and the mediastinal lymph nodes. The best course of
action would be to:
A. continue with chemoradiation therapy to a dose of 66 Gy.
B. proceed with definitive lobectomy and mediastinal lymph node dissection.
C. add one more course of chemotherapy and 10.8 Gy of radiation therapy, followed by
surgery.
D. recommend no further cancer therapy.

Correct answer is A. RATIONALE: Patients with progressive disease after preoperative
chemoradiation did not undergo surgery in either the SWOG 88-05 or Intergroup 0139 (surgical
arm) study due to the poor prognosis associated with these patients. Even though there is no
good standard approach for such patients, it is reasonable to continue full chemoradiation to
achieve a durable intrathoracic response and to minimize the risk of metastatic disease.
REFERENCE: Albain KS, et al. Lancet. 2009.

102. Which of the following radiation therapy regimens is most likely to provide the best local
control with minimal side effects in an 85-year-old man with a medically inoperable
2.5 cm peripheral NSCLC?
A. 50 Gy in 4 fractions using SBRT
B. 66 Gy in 33 fractions using 3D-CRT
C. 69.6 Gy in 58 fractions twice daily using 3D-CRT
D. 70 Gy in 30 fractions using IMRT

Correct answer is A. RATIONALE: SBRT with BED >100 Gy has been shown to provide >90%
local control with minimal side effects for a peripherally located stage I NSCLC. SBRT is
emerging as standard treatment for medically inoperable stage I NSCLC.

116. Which of the following adjuvant radiation therapy regimens is most appropriate for an
otherwise healthy 65-year-old man who has stage pT3N2M0 NSCLC with negative
surgical margins?
A. 50 Gy in 25 fractions
B. 60 Gy in 30 fractions
C. 70 Gy in 35 fractions
D. 74 Gy in 37 fractions

Correct answer is A. RATIONALE: With negative surgical margins and positive mediastinal
lymph nodes, the recommended postoperative radiation dose is 50 Gy in 25 fractions. Doses of
60 Gy and above are recommended for positive surgical margins.

144. A 3 cm carcinoma in the right upper lobe of the lung with a separate nodule in the same
lobe and aortopulmonary (AP) window adenopathy would be classified as stage:
A. T3N2.
B. T3N3.
C. T4N2.
D. T4N3.

Correct answer is B. RATIONALE: Based on the new, 2009 IASLC staging system, the
T stage for a separate tumor nodule in the same lobe as the primary tumor has been changed
from stage T4 to stage T3. For primary right lung cancer, the aortopulmonary (AP) window
lymph node is considered to have contralateral mediastinal lymph node involvement, and
therefore, should be stage N3 disease.

158. A 63-year-old man has a 6.2 cm squamous cell carcinoma metastatic to the ipsilateral
hilum without evidence of distant metastases. This patient’s disease is classified as
stage:
A. IIA.
B. IIB.
C. IIIA.
D. IIIB.

Correct answer is B. RATIONALE: Refer to the new AJCC staging system introduced on
January 1, 2010.

200. A 7.5 x 3 cm adenocarcinoma of the left upper lobe of the lung without pleural or
bronchial tree involvement would be classified as stage:
A. T1.
B. T2a.
C. T2b.
D. T3.

Correct answer is D. RATIONALE: Based on the new, 2009 IASLC staging system, the
T stage is divided by the tumor size: T1a = <2 cm; T1b = >2 cm but <3 cm; T2a = >3 cm but <5
cm; T2b = >5 cm but <7 cm; T3 = >7 cm. This information is new compared with the prior Tstage
system.

214. A 65-year-old man underwent lobectomy and mediastinal nodal dissection for stage
pT3N2 NSCLC with negative surgical margins. Which of the following treatment options
is most appropriate for this patient?
A. Close surveillance
B. Chemotherapy alone
C. Radiation therapy alone
D. Both chemotherapy and radiation

Correct answer is D. RATIONALE: Adjuvant chemotherapy improves survival in patients with
stage II to stage III NSCLC. Postoperative radiation therapy improves local control and
disease-free survival in patients with stage N2 disease. Concurrent chemoradiotherapy has not
been shown to further improve survival or local control in the adjuvant setting for patients with
stage N2 disease and negative surgical margins.

227. Which of the following clinical features is associated with pleural malignant
mesothelioma?
A. Approximately 80% of patients are male.
B. It more commonly occurs in patients less than 50 years old.
C. Chest pain occurs in less than 30% of patients.
D. Weight loss is a common presenting symptom.

Correct answer is A. RATIONALE: Eighty percent of patients with pleural malignant
mesothelioma are male, and patients commonly present with a pleural effusion associated with
breathlessness accompanied by chest wall pain (more than 60% of patients). The combination
of an unexplained pleural effusion and pleural pain should raise the suspicion of malignant
mesothelioma, even if the initial cytologic findings are negative. Weight loss and fatigue are
common later in the progression of pleural mesothelioma but are less common at presentation
(occurring in less than 30% of patients). Although a cytologic diagnosis can be made quickly,
malignant mesothelioma is usually not diagnosed until 2 or 3 months after the onset of
symptoms; delays of this length are especially frequent in centers in which the disease is
uncommon. Mesothelioma is occasionally discovered incidentally on routine chest radiography.
The risk of developing mesothelioma increases with age. Clubbing of the digits occurs in less
than 1% of cases.

240. After initial diagnosis, patients with malignant mesothelioma have a median survival time
of:
A. 06 months.
B. 12 months.
C. 18 months.
D. 24 months.

Correct answer is B. RATIONALE: The median survival of patients with malignant
mesothelioma from the time of diagnosis is 12 months. The prognosis is worse in male patients
and in patients with extensive disease, poor performance status, elevated white blood cell
counts, anemia, thrombocytosis, sarcomatoid histologic findings, or high standardized uptake
value ratios on PET scans. A worse prognosis is also associated with expression of certain
biochemical markers (cyclooxygenase-2 and VEGF), hypermethylation of the P16INK4a gene,
increased vascularity, and evidence of SV40 virus in the tumor.

253. Which of the following regimens is considered to be first-line chemotherapy for
malignant mesothelioma?
A. Cisplatin alone
B. Pemetrexed alone
C. Cisplatin and etoposide
D. Cisplatin and pemetrexed

Correct answer is D. RATIONALE: A combined first-line regimen using cisplatin and
pemetrexed is considered the gold standard for malignant mesothelioma. In a multicenter
phase III study involving 448 patients, those treated with pemetrexed plus cisplatin had a longer
overall median survival (12.1 months) than those treated with cisplatin alone (9.3 months) and
had an objective response rate (shrinkage of the tumor by at least 50%) of 41%. REFERENCE:
Journal of Clinical Oncology (J Clin Oncol). 2003;21:2636-44.

266. Which of the following outcomes was associated with the use of PCI after chemotherapy
for patients with extensive-stage SCLC, according to the EORTC 08993/22993
randomized trial?
A. The rate of symptomatic brain metastasis was reduced by 25%.
B. The benefit of PCI was limited to patients who had a complete response to chemotherapy.
C. There was no improvement in the overall survival rate.
D. The majority of patients died of CNS disease.

Correct answer is A. RATIONALE: EORTC 08993/22993 explored the use of PCI in patients
with extensive-stage small cell lung cancer (SCLC). Patients were only required to have a
response to chemotherapy and did not need to have a complete response. The primary
endpoint was the rate of symptomatic brain metastases, and this was significantly lower with the
use of PCI (40% vs. 15%). Extracranial progression occurred in 90% of patients. Also an
increase in overall survival was associated with treatment, on the order of 27% vs. 13% at
1 year. Patients were not required to have repeat brain imaging before the initiation of PCI,
which is a controversial point for many with regard to interpreting the results of the trial.
REFERENCE: Slotman B, et al. New England Journal of Medicine (N Engl J Med). 2007 Aug
16;357(7):664-72.

278. What percent of patients with SCLC present with limited-stage disease?
A. 10% to 20%
B. 30% to 40%
C. 50% to 70%
D. >90%

Correct answer is B. RATIONALE: A minority of patients with small cell lung cancer (SCLC)
present with limited-stage disease, and this proportion has not changed significantly over the
last 30 years (32.5% in 1973, 39.6% in 2002). REFERENCE: Govindan R, et al. Journal of
Clinical Oncology. 2006 October 1;24(28):4539-4544.

297. According to the Turrisi trial for SCLC, which of the following outcomes was associated
with twice-daily administration of radiation and chemotherapy?
A. The 5-year overall survival rate was 16%.
B. The rate of grade 3 esophagitis was 27%.
C. The rate of grade 3 pneumonitis was 40%.
D. The objective response rate was 40%.

Correct answer is B. RATIONALE: Compared to 45 Gy of radiation delivered in once-daily
fractions, 45 Gy of radiation delivered in 1.5-Gy twice-daily fractions resulted in improved overall
survival. Five-year overall survival was 26%, compared to 16% for the once-daily treatment
arm. Although survival improved with twice-daily radiation therapy, it came at the cost of
increased esophagitis (grade 3 esophagitis rate was 27% vs. 11% for once-daily radiation).
Elective irradiation of the supraclavicular regions was specifically forbidden. Objective response
rates approached 90% overall. REFERENCE: Turrisi AT, et al. New England Journal of
Medicine (N Engl J Med). 1999 Jan 28;340(4):265-71.

304. Which of the following chemotherapy regimens is most commonly delivered along with
chest irradiation for patients with limited-stage SCLC?
A. Carboplatin and etoposide weekly
B. Carboplatin and paclitaxel weekly
C. Cisplatin and etoposide every 3 weeks
D. Cisplatin and vinblastine every 3 weeks

Correct answer is C. RATIONALE: Based on the results of INT-0096, cisplatin and etoposide
given every 3 weeks for 4 total cycles (cycles 1 and 2 delivered at the start, and immediately
following the completion of radiation) has become the most commonly used regimen. As such,
it is listed as a category 1 recommendation by the NCCN guidelines for small cell lung cancer
(SCLC). REFERENCE: Turrisi AT, et al. New England Journal of Medicine (N Engl J Med).
1999 Jan 28;340(4):265-71.

311. According to Aupérin’s meta-analysis, which outcome was associated with the use of
PCI for patients who had limited-stage SCLC and achieved a complete response?
A. The 3-year overall survival rate was improved by 10%.
B. A clear dose response in overall survival was demonstrated.
C. The risk of CNS metastasis was reduced by 25% at 3 years.
D. The risk of neuropsychological sequelae was increased.


Correct answer is C. RATIONALE: Aupérin’s meta-analysis looked at individual data from
987 patients from 7 randomized trials comparing PCI to no PCI. The absolute improvement in
3-year overall survival was approximately 5% (from 15 to 21%). While larger total doses of
radiation appeared to improve local control, there was no improvement in overall survival. The
brain (CNS) failure rate was reduced from 58.6% to 33.3%, a 25.3% risk reduction. The effect
of PCI on neuropsychological sequelae could not be addressed by this meta-analysis.
REFERENCE: Aupérin A. New England Journal of Medicine (N Engl J Med). 1999 Aug
12;341(7):476-8.


318. A 65-year-old smoker has an extensive mediastinal adenopathy. A fine needle
aspiration of the subcarinal lymph node shows an extensive nuclear crush artifact in the
specimen. What is the most likely diagnosis?
A. Mesothelioma
B. Adenocarcinoma
C. Squamous cell carcinoma
D. Small cell carcinoma


Correct answer is D. RATIONALE: The presence of a nuclear crush artifact, while not
pathognomonic, is a highly suggestive morphologic feature for the diagnosis of small cell lung
cancer in this clinical setting. REFERENCE: Davenport R, et al. Acta Cytologica (Acta Cytol).
1990 Jul-Aug;34(4):502-4.

12. Which chemotherapy regimenis the MOST appropriate 1st line therapy for an epithelial mesothelioma?


(A) Cisplatin and Etoposide


(B) Cisplatin and Pemetrexed


(C) Carboplatin and Paclitaxel


(D) Carboplatin and Gemcitibine

Key: B Rationale:This randomizedtrial in mesothelioma was a major breakthrough in mesothelioma and should beknown by all physicians taken care of this disease. It was the 1st trial todemonstrate a benefit over cisplatin alone in the advanced setting.

18. What is the benefit of PCIfor a patient with extensive SCLC who had a good response to chemotherapy?


(A) Reduction in the incidenceof symptomatic brain metastases, but no effect on OS


(B) Reduction in the incidenceof symptomatic brain metastases, and an improvement in OS


(C) Reduction in the incidenceof asymptomatic brain metastases, and an improvement in OS


(D) No reduction in theincidence of symptomatic brain metastases, but an improvement in OS

Key: B Rationale:The role ofprophylactic cranial irradiation was evaluated by a randomized trial conductedin 186 patients with extensive small cell lung cancer who had had a response tochemotherapy. This study showed that prophylactic cranial irradiation reducesthe incidence of symptomatic brain metastases (HR0.27; 95% CI: 0.16–0.44, p< 0.001) and prolongs disease-free (14.7 weeks vs 12.0 weeks, p = 0.02) andoverall survival (6.7 vs 5.4 months, p = 0.003) Not only did they show adecrease in CNS metastases but also an improvement of overall survival at 1year (27% versus 13%). The cumulative risk of brain metastases at 1 year was40.4% in the observation arm and 14.6% in the therapy arm. Patients in thisstudy did not have routine CNS imaging. brain CT or MRI was done only ifpatients had symptoms of metastases.

87. Regarding the use ofadjuvant chemotherapy following surgical resection for lung cancer:


(A) there is an improvement inOS of 10%, based on the LACE meta-analysis.


(B) the combination ofcisplatin and vinorelbine demonstrated the largest benefit in OS.


(C) in the absence of nodalinvolvement, CALGB 9633 demonstrated an improvement in OS.


(D) the combination of carboplatin andpaclitaxel has the largest relative improvement in OS

Key: B Rationale:In the most recentupdate of CALGB 9633, which was the only phase III trial to test thecombination of carboplatin and paclitaxel, there was no difference in outcomesfor tumors > 3 cm without nodal involvement. There is a suggestion of animprovement in patients with tumors > 4 cm but this was based on anunplanned subset analysis. The survival benefit of adjuvant chemotherapy basedon the LACE meta-analysis was 4% with the largest improvement seen in patientstreated with cisplatin and vinorelbine.

134. Regarding the roleof surgery following chemoRT for stage III non-small cell lung cancer:


(A) 60 Gy is requiredto demonstrate an improvement in OS with surgery.


(B) surgery improves OSin all resectable patients based on level 1 evidence.


(C) a lobectomyimproves local control and OS based an unplanned subgroup analysis of theintergroup trial.


(D) a pneumonectomydemonstrates a local control and OS for left side tumors based an unplannedsubgroup analysis of the intergroup trial.

Key: C Rationale: The Intergroupexperience published by Albain demonstrated a significant improvement in localregional control with the addition of surgery following chemo radiotherapy overradiotherapy alone although this did not translate into a survival advantagerelated to the mortality of surgery – primarily the right sidedpneumonectomies. An unplanned analysis of this data suggested that patientsundergoing a lobectomy had less morbidity/mortality and subsequently this groupof patients had a survival advantage.

149. For a patient withextensive stage SCLC receiving cisplatin/etoposide, the expected response rate(RR) and the median OS associated with this regimen is:


(A) RR of 30-50% andmedian OS of 4-5 months. (B) RR of 30-50% andmedian OS of 7-9 months. (C) RR of 60-80% andmedian OS of 7-9 months. (D) RR of 60-80% andmedian OS of 18-24 months.

Key: C Rationale: Chemotherapy is the standardtreatment for extensive-disease small cell lung cancer. The combination ofcisplatin and etoposide (EP) or carboplatin and etoposide (EC) up to 4–6 cyclesremains the most widely used regimen, with an ORR ranging from 60% to 80%.However, the median OS is still 7– 9 months, and only 2% of patients survivefor 5 years.

155. Regarding limitedstage SCLC, chemoRT demonstrated:


(A) a 5.4% improvementin OS compared with RT alone.


(B) a 5.4% improvementin OS compared with cisplatin-based chemotherapy alone.


(C) a 5.4% improvementin LRC, but no improvement in OS compared with RT alone.


(D) a 5.4% improvementin LRC, but no improvement in OS compared with cisplatin-based chemotherapyalone.

Key: B Rationale: Due to highlocoregional failure rates after chemotherapy alone, thoracic radiation incombination with chemotherapy was investigated for patients with limited stagedisease. Several randomized studies compared chemotherapy alone to chemotherapyand radiation.

163. What is theoptimal cycle of chemotherapy to start concurrent thoracic radiation for ahealthy patient with limited stage SCLC?


(A) 2


(B) 3


(C) 4


(D) 5

Key: A Rationale: The current standard ofcare is concurrent chemotherapy and radiation, with radiation being deliveredin the first two courses of chemotherapy. LRC and survival is better withconcurrent rather than sequential therapy but at the cost of increasedtoxicity. Sequential therapy is acceptable for patients who may not toleratethe added toxicity of concurrent therapy or have large tumor volumes and/orpoor pulmonary function. Volume reduction with chemotherapy may allow forsparing of normal tissue and better therapy tolerance.

190. Regarding RTOG0617, which evaluated (1) the benefit of an additional 14 Gy to the standarddose of 60 Gy and (2) the benefit of the addition of cetuximab to thechemotherapy?


(A) No difference in OSwas seen between all four treatment arms.


(B) There was an OSbenefit in the 60 Gy arm with either chemotherapy combination.


(C) There was an OSbenefit in the 74 Gy arm with either chemotherapy combination.


(D) The OS benefit inthe 74 Gy arm was limited to chemotherapy without cetuximab.

Key: B Rationale: The RTOG 0617 trial wasa phase III study with a double randomizationcomparing (1) 60 vs. 74 Gy, andchemotherapy consisting of carbo/paclitaxel alone vs. carbo/paclitaxel withcetuximab. The median survival times and 18-month OS rates for the 60 Gy and 74Gy arms were 28.7 vs 19.5 months, and 66.9% vs 53.9% respectively (p <0.001). Unfortunately, and surprisingly, not only was there no benefit toescalating doses to 74 Gy, this arm did statistically worse.

218. Which of thefollowing clinical stages would have been eligible for the SWOG 9416/INT0160study of superior sulcus tumors?


(A) T2N1


(B) T2N2


(C) T3N2


(D) T4N1

Key: D Rationale: The SWOG 9416/INT 0160was a phase II study examining neoadjuvant chemotherapy and radiation forpatients with superior sulcus tumors. Eligible stages were T3-4N0-1. Patientswere treated to 45 Gy with concurrent cisplatin and etoposide. The rate ofcomplete resection was 76%. 5 year overall survival was 44%.

228. Regardingthymomas/thymic carcinomas:


(A) pericardialinvasion is considered unresectable disease.


(B) there is no rolefor chemotherapy in the management of thymomas.


(C) involvement ofmediastinal lymph nodes is considered stage IVB disease.


(D) thymoma-associatedmyasthenia gravis is readily reversed with surgical resection.

Key: C Rationale: In the modified Masaokastaging system, LN+ disease is considered stage IVB. Other statements areincorrect. Pericardial invasion is still considered resectable, often done withinduction chemotherapy followed by maximum resection. Surgery does not reverseMG related symptoms right away, and may require continued therapies. Thymomasare sensitive to platinum agents, and CAPP is often employed as inductionchemotherapy prior to surgical resection for bulky disease

245. Regarding the roleof adjuvant RT following complete surgical resection (negative margins) in apatient with NSCLC with evidence mediastinal lymph node involvement?


(A) There is no rolefor RT following adjuvant chemotherapy.


(B) RT should bedelivered sequentially to decrease the morbidity.


(C) The total doseshould be 60 Gy since the tumor bed has been devascularized.


(D) Chemotherapy shouldbe administered concurrently to get the best local control and OS.

Key: B Rationale: Based on retrospectiveanalyses though SEER (Lally) and ANITA, PORT has demonstrated substantialbenefits in the adjuvant chemotherapy era. All the data demonstrating a benefitwas seen in the sequentially treated patients suggesting high complicationrates related to concurrent approach (ECOG). Doses > 54 Gy have been shownto increase complications (Dautzenberg and Machtay).

262. Separate tumors inthe same lobe are considered to signify what AJCC stage of disease? (A) T2 (B) T3 (C) T4 (D) M1a

Key: B Rationale: Two tumor nodules inthe same lobe are considered T3 disease. Two tumor nodules in separate butipsilateral lobes are T4 disease, and a separate tumor nodule in acontralateral lobe is considered M1a disease.

264. What was the5-year OS rate in the SWOG 9416/INT0160 trial for superior sulcus tumors? (A) 32% (B) 44% (C) 56% (D) 68%

Key: B Rationale: The SWOG 9416/INT 0160was a phase II study examining neoadjuvant chemotherapy and radiation forpatients with superior sulcus tumors. Eligible stages were T3-4N0-1. Patientswere treated to 45 Gy with concurrent cisplatin and etoposide. The rate of completeresection was 76%. 5 year overall survival was 44%.

288. Which actiondecreases the risk of radiation pneumonitis?


(A) Hypofractionation


(B) Continued smoking


(C) Concurrentchemotherapy


(D) Concurrentimmunotherapy

Key: B Rationale: Continued smoking aftertreatment is associated with a decreased risk for radiation pneumonitis,possibly because it suppresses radiation-induced inflammation. Chemotherapy,immunotherapy and hypofractionation could increase the likelihood ofpneumonitis, not decrease it.